Jump to content

Wikipedia:Reference desk/Science: Difference between revisions

From Wikipedia, the free encyclopedia
Content deleted Content added
Line 577: Line 577:
Someone told me that there is (or could be) a whole universe in my thumb. Do you know what they mean by that? --[[User:Anthonygiroux|Anthonygiroux]] ([[User talk:Anthonygiroux|talk]]) 13:19, 7 July 2008 (UTC)
Someone told me that there is (or could be) a whole universe in my thumb. Do you know what they mean by that? --[[User:Anthonygiroux|Anthonygiroux]] ([[User talk:Anthonygiroux|talk]]) 13:19, 7 July 2008 (UTC)
:I personally have no idea. Maybe it was an [[idiom]]? — <font face="rage italic" size="4.5px">'''[[User:Cyclonenim|''CycloneNimrod'']]'''</font>&nbsp; <sup><font face="Tahoma" size="1">[[User_talk:Cyclonenim|Talk?]]</font></sup> 13:24, 7 July 2008 (UTC)
:I personally have no idea. Maybe it was an [[idiom]]? — <font face="rage italic" size="4.5px">'''[[User:Cyclonenim|''CycloneNimrod'']]'''</font>&nbsp; <sup><font face="Tahoma" size="1">[[User_talk:Cyclonenim|Talk?]]</font></sup> 13:24, 7 July 2008 (UTC)

==How could this be?==
[[Tomoji Tanabe]] is 112 and seems to be extremely healthy. My grandmother is 86 and is in hospital sick. Higher ages seem to be more common in females, so how was this jumble caused? [[Special:Contributions/208.76.245.162|208.76.245.162]] ([[User talk:208.76.245.162|talk]]) 13:31, 7 July 2008 (UTC)

Revision as of 13:31, 7 July 2008

Welcome to the science section
of the Wikipedia reference desk.
Select a section:
Want a faster answer?

Main page: Help searching Wikipedia

   

How can I get my question answered?

  • Select the section of the desk that best fits the general topic of your question (see the navigation column to the right).
  • Post your question to only one section, providing a short header that gives the topic of your question.
  • Type '~~~~' (that is, four tilde characters) at the end – this signs and dates your contribution so we know who wrote what and when.
  • Don't post personal contact information – it will be removed. Any answers will be provided here.
  • Please be as specific as possible, and include all relevant context – the usefulness of answers may depend on the context.
  • Note:
    • We don't answer (and may remove) questions that require medical diagnosis or legal advice.
    • We don't answer requests for opinions, predictions or debate.
    • We don't do your homework for you, though we'll help you past the stuck point.
    • We don't conduct original research or provide a free source of ideas, but we'll help you find information you need.



How do I answer a question?

Main page: Wikipedia:Reference desk/Guidelines

  • The best answers address the question directly, and back up facts with wikilinks and links to sources. Do not edit others' comments and do not give any medical or legal advice.
See also:


July 1

Adultration

how to cure adultration in products purchased from the market? —Preceding unsigned comment added by Jasleen302 (talkcontribs) 12:00, 28 June 2008 (UTC)[reply]

Best to avoid buying them in the first place. Check your product carefully before you buy, and insist on a pure product. Buying this sort of stuff will just encourage more adulteration. What products are you having trouble with. Graeme Bartlett (talk) 01:57, 1 July 2008 (UTC)[reply]
By having an effective network of inspectors, checking for adulteration, and with the power to take action when it is found? (if I have understood the question correctly?)87.102.86.73 (talk) 09:58, 1 July 2008 (UTC)[reply]
This has been a problem for a long time, and it ebbs and flows with waves of government regulation and deregulation. The classical (fictionalized) work on this is Upton Sinclair's muckraking novel The Jungle.
Atlant (talk) 16:57, 1 July 2008 (UTC)[reply]

Unmanned Aircraft

I was watching a documentary, Regulus - The First Nuclear Missile Submarines, after WW2, in that it states how they managed to make the missile from remote go up and down, right and left, and eventually land. More information: [1]. Anyways if can control a missile from remote, why has there been no unmanned aircraft yet? Like a remote control one? I mean with the GPS and satellite technology it is possible. Is the United States Forces holding back on something they have in secret? --69.156.94.136 (talk) 01:17, 1 July 2008 (UTC)[reply]

There are unmanned aircraft. They don't even need to be remote controlled, they can be controlled by computer AI. Unmanned combat aircraft are probably the future of air warfare. Bombers and attack aircraft will be first. Followed by fighter aircraft.ScienceApe (talk) 01:29, 1 July 2008 (UTC)[reply]
It's not a secret, and it's not being held back. The MQ-1 Predator is a remotely-controlled unmanned aerial vehicle that fills a variety of roles, including precision bombing. It has been flying armed missions since at least 2001. TenOfAllTrades(talk) 02:24, 1 July 2008 (UTC)[reply]
Coming soon to an administration near you: Ender's Game.
Atlant (talk) 16:55, 1 July 2008 (UTC)[reply]
Modern commercial airliners with an autopilot are virtually unmanned aircraft, see this section Autopilot#Modern autopilots, and potentially only need a pilot for taxiing. Human pilots are almost a back up system to the aircraft autopilot and to provide a sense of security to the passengers. Tobyc75 (talk) 17:59, 2 July 2008 (UTC)[reply]

Freezing Cheese: Good or Bad

I wanted some input on freezing cheese. I'm taking a five hour flight and want to take a two pound block of cheese with me. I know that many people say, cheese shouldn't be refrigerated because it inhibits the life, and flavor inside the cheese, But hey I'm an American. And I know that freezing turns the moisture inside items into sharp shards, basically perforating what ever your freezing from the inside. But is freezing cheese GOOD or BAD? —Preceding unsigned comment added by Semi-smart (talkcontribs) 03:34, 1 July 2008 (UTC)[reply]

Freezing cheese is mostly successful, hard or soft, but not ricotta, so anything close and without moisture traps inside. Julia Rossi (talk) 07:07, 1 July 2008 (UTC)[reply]
Freezing cheddar is no good, it destroys the texture and it all goes crumbly. Parmesan powder freezes OK though. For a short flight like this, just pre cool it in the fridge, and then wrap it in something insulated like bubble wrap to keep it cool. Graeme Bartlett (talk) 07:13, 1 July 2008 (UTC)[reply]
But freezing cheddar is fine if you're going to use it in cooking, since the texture won't matter [2]. My Googling suggests that this is true of a lot of cheeses... in applications where texture doesn't matter, freezing is usually fine. --Allen (talk) 07:18, 1 July 2008 (UTC)[reply]
I have successfully frozen Stilton style blue cheese, flavour and texture were unaffected after 3 months. Richard Avery (talk) 07:25, 1 July 2008 (UTC)[reply]
I agree with Graeme Bartlett that the best way to deal with this would be to just cool it and keep it cool by insulating it. A simple way to do this would be to buy a small cooler -- I've seen ones that are about the size of a small handbag -- and sticking a couple of ice packs in with the cheese. (The "ice pack" article appears to deal only with the medical ice packs, but what I mean is the thing that is also called a "freezer block" or "ice brick", a rigid plastic brick filled with water that you can freeze and then use to keep things cool, the mainstay of coolers all over the world. (Can it be that we really don't have an article for them, or am I just completely blanking on the terminology here?)) Depending on where you are and how paranoid they are about you using your cheese to destroy the plane in midair, taking that with you as carry-on might be problematic, but you could just stick it in your bag and check it.
That said, a cheaper and more low-tech approach would be simply wrapping the refrigerated cheese tightly in newspaper. It makes a pretty good insulator, and you could always stick in an ice pack with the cheese to make sure it stays cool, if you're worried about that, but the newspaper by itself will probably do the trick, if the cheese is cool to begin with. You can probably get some extra mileage out of the newspaper by sticking it in the fridge with the cheese so it's also cool. As long as the newspaper is dry (water conducts heat very well), it'll work just fine. -- Captain Disdain (talk) 11:02, 1 July 2008 (UTC)[reply]

Why don't you test it out? Try freezing a bit and seeing whether it survives. I would be interested in hearing your results. Plasticup T/C 13:01, 1 July 2008 (UTC)~[reply]

I say screw it. The flight is only 5 hours pal, leaving cheese out of the fridge for that long probably won't be that bad for it. Moreover, if it's 2 pounds of cheese it should hold a temperature for a while. I'm all for captain disdain's strategy of insulating the cheese. But for the record, remember that if you're flight is international they're probably going to take the cheese at customs. --Shaggorama (talk) 16:46, 1 July 2008 (UTC)[reply]
When we returned from France, we brought back a variety of cheeses, probably mostly of the semi-dry styles. They were wrapped well in butcher paper at the cheese shop and we then closed the cheese shop's plastic bag as well as we could and chucked them in the middle of the clothes in our luggage. Between the insulation of the clothing and the likely cold baggage hold of the plane, they survived the ten-ish hour trip just fine. And even though the cheeses were from (gasp!) France, we declared them at American customs and were still allowed back into the country, cheeses and and all. (The preceding does not constitute legal advice; it just recounts Wikipedian's one anecdote.)
Atlant (talk) 16:53, 1 July 2008 (UTC)[reply]
This might help clarify matters. [3] But be sure to print out a copy to hold under their noses in case someone makes a fuss. OR: I've encountered customs officials anywhere from waving people through to trying to declare that permitted goods were not so. (Chocolate could cause all sorts of delays in NY for a couple of years while back, but that seems to have dissipated. I assume some overzealous supervisor got retired.) As for transporting the cheese, if you have the spare luggage capacity you might freeze a couple of those soft Ice packs you can buy at drugstores or pharmacies or the like. If you are squeezing for space and weight a wet T-shirt or towel might do the trick to prevent your cheese from suffering in case your luggage ends up baking in the summer sun for a couple of hours instead of going directly from the cool terminal to the cold cargo hold. NB: make sure to unpack immediately once you're home and air out the luggage to prevent moisture damage. Bubble wrap suggested above would additionally prevent your cheese from turning into cheese crumble, thanks to baggage handling. Remember you won't be able to take it as carry-on [4]. 80.171.254.148 (talk) 01:33, 3 July 2008 (UTC)[reply]
The recommendation for hard cheese is probably good for customs reasons as well; you want to avoid bringing back anything like Abbaye de Citeaux, Chevrotin and Chabichou du Poitou, "the forbidden cheeses", which have a moisture content that makes U.S. Customs uncomfortable. (What is specifically banned is raw milk cheese aged less than 60 days, or aged more than 60 days but having a moisture content of more than 67% water). This is sad for Americans, because, in the words of the Amateur Gourmet, the "high moisture content gives a creamy, luxurious texture and the raw milk lends it complexity and depth." You will have to enjoy these abroad.... - Nunh-huh 02:59, 3 July 2008 (UTC)[reply]

QAM query

Hello to all Please if some one can guide me about data rates of different catagerizes of QAM that is the data rate of 8-QAM, 16-QAM, 32-QAM, 64-QAM, 128-QAM, 256-QAM

thanks —Preceding unsigned comment added by Arsalan 80 (talkcontribs) 04:29, 1 July 2008 (UTC)[reply]

The different rates will depend on what bandwidth in general you want. For particular applications such as digital television or ADSL the appropriate standard will specify the rates to select from. With 8-QAM you will get 3 bits per symbol, 16-QAM will give you 4 bits per symbol, and so on till 256-QAM with 8 bits per symbol. Graeme Bartlett (talk) 06:18, 1 July 2008 (UTC)[reply]

statistics

what the the distinction between simulation,models,and experiments? —Preceding unsigned comment added by 72.75.74.106 (talk) 11:46, 1 July 2008 (UTC)[reply]

Experiments are tests on real-world things that yield real-world data. Simulations are more like thought-experiments. They are involve creating a model to imitate the real thing. Plasticup T/C 11:58, 1 July 2008 (UTC)[reply]
In my experience, a model is something that is fed into a simulation to give it the information needed to simulate, so to speak. Said in another way, the model is the abstraction and the simulator takes that abstraction and performs calculations on it to produce an expected result. The Monte Carlo method is a good example of this. The model is the input and the simulation is the action taken on it and the final result. In common parlance, they are often used interchangeably. An experiment is a totally different thing. It is an empirical study of something. Rather than having a known model, a series of observations are taken and analysed. In some cases, a model fit can be used to test a hypothesis. It is important to note that the experiment is totally based on observation. Simulation and modelling can be used to create a hypothesis, but experimentation is usually necessary to reject or not reject a hypothesis. Gjmulhol (talk) 12:06, 1 July 2008 (UTC)[reply]
A model can be more than just an idea. In biology, a model could be a live mouse, used to "simulate" a human for a particular purpose. Someguy1221 (talk) 03:22, 3 July 2008 (UTC)[reply]

Help with electronics

redirected from Wikipedia:Reference desk/Computing#Help with electronics

I have just begun building a guitar pedal, but no have no electronics expereience, but a good deal of knowledge (theoretical knowledge) of the subject.

I have become stuck where the schematic tells me to connect to ground.

I am soldering to a sheet of stripboard, and it has a thin copper track running down both sides of the board, but with no holes in them; Is this where i should connect grounded connections to? And do i connect all grounded connections to the one place, or should i cut the track so as to separate them from one another?

Any help would be much appreciated.

Thank you!! —Preceding unsigned comment added by 212.46.129.226 (talk) 09:33, 1 July 2008 (UTC)[reply]

If this is a standalone pedal (not powered by a wall outlet), it is very possible that you do not have ground inside the footpedal at all. In that case, I would assume that you are using a 1/4" plug from the guitar/amp and assume that the shielding of that connector is ground. So, attach there. If, instead, you are powering this from a wall outlet, ensure you are using a grounded plug (three prongs in the U.S.). Then, you will have ground where the power is soldered onto the board. Because this is an audio device, be wary of ground loops. If you ground your pedal and plug it into one outlet, then plug the guitar/amp into another outlet, it is possible that they do not share the same ground. You can easily get a hum or buzz on the speakers. So, inside the pedal, ensure you connect the ground from your power input to the ground on the shield of the cable from the guitar/amp. Then, you will joining the possible difference in grounds and eliminating the buzz. -- kainaw 12:54, 1 July 2008 (UTC)[reply]

Additional Help with Electronics

Thanks to kainaw for answering my previous question about grounding, but i still have some queries.

When you say to connect to the ground from the guitar jack, do you mean the input or the output jack?

And does it matter whether i connect all the grounded connections directly to the ground terminal of the jack, or if i connect them all (Including the ground from the jack) to the copper track at the side of the stripboard?

Thanks again for your help —Preceding unsigned comment added by 212.46.129.226 (talk) 13:41, 1 July 2008 (UTC)[reply]

"Ground" is often used in an imprecise way. The two most common usages are as "Ground"/"Earth" (yes I know this has reused "Ground") for the purpose of one or more of safety and screening/shielding and as 0V. Eventually Ground/Earthwants to get connected to a physical Ground - eg a copper rod stuck into the planet's soil. For this to work well as a safety connection, it wants to be well connected "in all directions". For it to work well as screening, you don't want current to flow through it, so only one connection to each bit of ground should be made, ideally in a star configuration but at worst as a tree configuration branching out from the trunk where the copper rod is. The other common usage informally means 0V. This has come about since often the 0V terminal of a PSU gets connected to ground to "stop it floating". This often creates loops and current flows in screening circuitry. With your pedals etc, you want all the 0V connected to each other once (no loops). A starter would be to see if, without your circuit in place the screen of the Inlet jack was connected somehow to the screen of the Outlet jack. If not, connect your ground signal to BOTH outers, otherwise choose one - the most convenient. There is an element of art rather than science about getting ground/0V/earth/screening right. -- SGBailey (talk) 13:52, 1 July 2008 (UTC)[reply]
To expand on SGBailey's point, "ground" in the context in which you're asking means "the reference voltage" (which we conventionally then call "0 volts"). All other signals in the circuit are measured "in reference" to this point, "the ground". For your guitar effects pedal, both the input and output jacks each have an outer terminal which should be connected to "the ground" (reference voltage) in your circuit.
Odds are that your guitar effects pedal should be designed to operate solely from battery power. Otherwise, any connection to mains/line power, even through a power supply/wall wart, is likely to cause a ground loop and that can induce your guitar amplifier to produce noise, hum, and other nasty stuff.
Atlant (talk) 16:42, 1 July 2008 (UTC)[reply]
A good book you might want to look into is "Do-It-Yourself Projects fo Guitarists" by Craig Anderton. I used to know a great website too, but I can't find it right now. --Shaggorama (talk) 16:09, 4 July 2008 (UTC)[reply]

Figure of the Earth image

I can't find this anywhere. I would like to know if there is an image which shows the figure of the earth without the water on it. I.e. only the rock part of the earth. Does such an image exist?  — Adriaan (TC) 13:44, 1 July 2008 (UTC)[reply]

Bathymetry, [5]? -- Coneslayer (talk) 13:46, 1 July 2008 (UTC)[reply]
No, not what I'm looking for. I was wondering if there was a pic in true colour of the earth, just with the water edited out. Something like an artist's impression of what the earth would look like if all the water suddenly disappeared. Like a 3D pic of an object supposed to be the earth - but without water.  — Adriaan (TC) 15:52, 1 July 2008 (UTC)[reply]
How about Mars? --Shaggorama (talk) 16:40, 1 July 2008 (UTC)[reply]
There's a 2D texture here. You could create a 3D image out of it quite easily with Xplanet. — Matt Eason (Talk &#149; Contribs) 20:03, 1 July 2008 (UTC)[reply]

Space program benefits

In reading about the ancillary benefits of NASA's space program in articles like this and this, it strikes me that most the advances that are mentioned aren't dependent on the space travel part of the equation. Rather, they were inspired or triggered by space travel, but could just as easily have been developed for the same cost for a land-based application. For example, water filtration was (according to the NASA site above) developed in part for the Apollo program, but it's not as though it couldn't have been developed for non-space applications. Are there practical benefits to the space program, readily apparent to regular folks, that could only come with space travel? jeffjon (talk) 14:28, 1 July 2008 (UTC)[reply]

Define 'regularly apparent to regular folk'. There are thousands of experiments in space, many of which will have furthered our understanding of the world and how things react in given situations. Do you just want a list of products that use techniques based on the space program? Or do wooly benefits such as "improving the knowledge of mankind" and "continuing the exploration of the universe"? Personally I find that when people start to try to qualify the value of Nasa based on products/technological developments overlook the strongest argument for the space-program - that there's a universe out there that holds limitless possibilities and the more we go out there and the further we venture the more interesting things we will find. Of course this is from an exploratory enthusiast who supports the cause, certainly there are plenty of worthy groups vying for already scarce government funding. 194.221.133.226 (talk) 15:02, 1 July 2008 (UTC)[reply]

I'm with you; I think exploration for sake of pure discovery is motivation enough. My wife isn't convinced, so I was trying to come up with a more tangible benefit that she receives for that almost 1% of every dollar she pays in taxes, whether it's a product, a health benefit, a scientific advance with other implications, etc. jeffjon (talk) 15:10, 1 July 2008 (UTC)[reply]

IMHO the scientific benefits are good enough. But to counter the "they totally could've been developed anyway" argument, I guess the question is why. Why would any of that be built? Why would the US build a water filter when we've got good tapwater? Necessity is the mother of invention. We need a way to recycle water in space. It also happens to work really well on Earth? We should sell that to the third world and people who are superstitious about tap water! As the first source you cite mentions, if you look at a lot of the products, most of them are saying "NASA was doing this obscure scientific thing and it totally improved this product by leaps and bounds!" Thats pretty damn impressive, and I consider it a good testament to the power of a good R&D budget. EagleFalconn (talk) 15:24, 1 July 2008 (UTC)[reply]

I wouldn't expect R&D for space to be any more likely than anything else to have things that help in unexpected ways. If NASA didn't invent water filtration, maybe it wouldn't have occurred to anyone, but, by the same token, maybe there's something that hasn't occurred to us because we weren't spending that money on R&D for something of more obvious practical significance. Doing R&D for something practical has the added advantage of helping in expected ways. For those of you who think NASA does pull its weight, how much of your money do you donate to it? You don't think it's only worth it if it's money the government takes from you forcefully, do you? I'm currently saving up for college, but when I'm done with that, I plan on donating to a microcredit group, like Unitus. I think they have more practical significance than NASA. — DanielLC 15:49, 1 July 2008 (UTC)[reply]

Satellites enable my contributions to Wikipedia. There's your human benefit right there. ;-) Plasticup T/C 17:11, 1 July 2008 (UTC)[reply]
That's a bit of a red herring, isn't it? I don't make voluntary cash donations to my local water treatment plant or the city subway system, but I still think it's worthwhile for those projects to receive funding. If you want to talk about research, in the United States good solid research is carried out or funded by government agencies from the Department of Energy to the Department of Veterans Affairs; I don't make donations to those organizations either. TenOfAllTrades(talk) 17:26, 1 July 2008 (UTC)[reply]

Global Positioning System saves a lot of time and needless erring around. LINEAR is another point. And then, for people like me, it's simply interesting and sometimes even exciting to look at real pictures from space. And when I saw Neil Armstrong live on TV stepping on the moon that is something to remember. 77.3.134.249 (talk) 20:27, 1 July 2008 (UTC)[reply]

Though I am certainly no expert, I have read in the past that zero gravity experiments have been done on cultures of bacteria. These have shown unique characteristics. I also remember evidence of a super-bug, but I don't remember where that is from. Gravity is vital to life, it is important that we know how plants and animals react in such an environment. From a materials perspective (I am a materials scientist), some crystals and structures can only be formed in a zero-g environment. If extra-planetary colonization ever takes place, it would be good to know how to take advantage of these new, potentially revolutionary structures. Gjmulhol (talk) 11:55, 2 July 2008 (UTC)[reply]

Note that just because spinoff products are formed does not necessarily mean that NASA is a good way to get them. If you give me a billion dollars I'm sure I could get you some tangible benefit for it—but would that be a better approach to said products than, say, letting them develop on the free market? That is—the return on R&D in terms of secondary benefits is going to be pretty small, most likely, and not very efficient.
In my opinion a better argument focuses on the BIG, non-spinoff benefits. GPS. Satellite communications. Military advances in rocketry. Advances in computing, robotics, materials science, etc. Water filtration? Not impressive. You could easily have gotten something like that without NASA. But GPS? Probably not. (And while "knowledge for its own sake" works great for people who would post on a board like this, it's not that convincing for the common man. Very little of that knowledge for its own sake has any payoff, and there's a lot of different types of "knowledge for its own sake" that one could be funding, much of it for a lot cheaper than space budgets.) --98.217.8.46 (talk) 18:37, 3 July 2008 (UTC)[reply]

physics

Question: When we throw a ball on the wall in a particular direction, then why does the ball comes back to us in some other direction ?'' —Preceding unsigned comment added by 61.2.190.79 (talk) 16:33, 1 July 2008 (UTC)[reply]

Start with Reflection (physics), but the most basic principle is that "the angle of incidence equals the angle of reflection". And remember, gravity is constantly working on the path of the ball.
Atlant (talk) 16:37, 1 July 2008 (UTC)[reply]
More specifically, for a ball: Deflection (physics). This article could do with expansion though ....Jdrewitt (talk) 16:53, 1 July 2008 (UTC)[reply]
More simply: It would take more energy to send the ball back in the direction it came from. Mac Davis (talk) 16:57, 1 July 2008 (UTC)[reply]
Well, yes and no; it would just take a corner cube, but gravity is still affecting the track of the ball (which is probably what you were referring to in the need for "more energy").
Atlant (talk) 19:48, 1 July 2008 (UTC)[reply]
Also, it is very difficult to throw a ball with zero spin. If you are familiar at all with the sport of tennis you will know that depending on the surface the ball is striking, the spin can change the direction dramatically. This is particularly evident if you use a hard rubber ball on a rough surface (e.g., a lacrosse ball on asphalt). The spin will alternate and dampen until the ball rolls flat. Thus, you see a low, long bounce followed by a short, high bounce, and on and on until the ball flies with negligible spin or it simply starts rolling. Gjmulhol (talk) 11:52, 2 July 2008 (UTC)[reply]

Endorphin as drug

Why not simply inject endorphin instead of some artificial substance if the purpose is to feel right? —Preceding unsigned comment added by Mr.K. (talkcontribs) 18:42, 1 July 2008 (UTC)[reply]

And where would you obtain the natural endorphins? You can't just buy it. And if you synthesized it, it would then be an artificial rather than "natural" substance. The closest analogue is morphine, which is routinely injected during childbirth and other painful periods. ~Amatulić (talk) 18:53, 1 July 2008 (UTC)[reply]
I'm under the impression that one would have to inject the endorphin(s) directly into the brain, which sounds rather messy. --Several Times (talk) 20:13, 1 July 2008 (UTC)[reply]
This is a common question with relation to most centrally acting drugs (serotonin vs SSRIs, norepinephrine vs salbutamol, etc). The most common problem with endogenous substances is that they won't cross the blood-brain barrier, other problems are listed in the pharmacokinetics article (absorption, distribution, metabolism and excretion). --82.21.25.219 (talk) 08:21, 2 July 2008 (UTC)[reply]
Also note that administering natural endorphins long-term would likely cause the same problems as other such drugs. A tolerance would develop, requiring more and more to get the same effect. And addiction would occur, so people would feel sick if the endorphin supplement was ever stopped. StuRat (talk) 15:42, 2 July 2008 (UTC)[reply]

Insulators for copper wiring

I'm wondering if there is any way to accuratly measure the resistivity of a substance...is there a way to get a quantitative value for resistivity in substances that could potentially be used to insulate basic copper wiring? 24.34.168.154 (talk) 18:43, 1 July 2008 (UTC)Fiziks[reply]

Using a Megger? Fribbler (talk) 18:46, 1 July 2008 (UTC)[reply]
A megger is a good answer if you're interested in the performance of the insulator at relatively high voltages (~1KV), but at lower voltages, one would use a nanoammeter, a voltage source, and some careful experiment design including guarding. Keithley Instruments is a typical vendor of nanoammeters. And here's a good article from EDN about proper techniques: [6].
Atlant (talk) 19:43, 1 July 2008 (UTC)[reply]
A Megger will measure the insulation resistance not the material resistivity. There are standard ways to measure resistivity: [7]


July 2

Infantry vs. tank

[This is really eight related questions, some of which may share answers.] Consider either (a) one or (b) eight soldiers armed with normal-caliber machine guns and an arbitrary amount of any kind(s) (armor piercing rounds?) of ammunition for them. They are free to walk the top of a tall concrete wall that is, say, 75m on a side. Trapped inside the wall on open ground is either (a) one unarmed enemy tank or (b) one enemy and one friendly tank, armed only with their main cannons. All tanks are modern main battle tanks; the unarmed tank may be considered to be repairing supposedly-minor damage to its anti-personnel weapons in (vain) hopes that it may return fire, and the armed tanks fire only at each other. How may the infantry in each of the four situations disable or destroy the enemy tank (a) in the least time or (b) using the least ammunition?

The purpose of the first bifurcation is to increase the infantry's effectiveness in the situation where there is a friendly tank, as well as to allow them to surround the enemy. The purpose of the second is to distinguish between an enemy that is fighting (albeit not against the infantry) and one that is doing nothing but trying to evade/weather fire. The purpose of the third is obvious, but in the case where there is a friendly tank present the "least ammunition" option should be taken to require some material contribution to the battle. I would also be interested in hearing about the mechanism by which (a large amount of) machine gun fire damages the high-grade armor against which it is not frequently used. Thanks for the myriad answers. --Tardis (talk) 01:11, 2 July 2008 (UTC)[reply]

Good luck. Modern tank armor (e.g. Chobham armor) is really, really tough. I'm no expert, but I doubt that anything fired from a machine gun would have a chance of doing any real damage. You're probably better off having these guys break out their shovels and cover the trapped tank with dirt. Clarityfiend (talk) 06:49, 2 July 2008 (UTC)[reply]
They could try and open the hatch and shoot the people inside. Trying to damage the armour would be in vain, I imagine. --Tango (talk) 14:04, 2 July 2008 (UTC)[reply]
Yeah- small arms fire against modern armor would be useless, no matter what ammo you're using. You'd need to get the hatch open somehow. Trap the tanks somehow, and wait for the people inside to get hungry? I don't know. Friday (talk) 14:39, 2 July 2008 (UTC)[reply]
75m square? What sort of ground? Discharge five 40 tonne road tankers of diesel into the area (so 3cm deep without bothering to do the maths) and ignite it? The thermal conduction of armour is pretty high and baking the people in it cannot be too hard. --BozMo talk 14:55, 2 July 2008 (UTC)[reply]
Best you could with small arms would be to try to blind and deafen them by destroying the optics and antennas. --—— Gadget850 (Ed) talk - 14:58, 2 July 2008 (UTC)[reply]
Even with an arbitrary amount (and duration; give the tanks infinite fuel if needed) of sustained fire? Surely something would happen to the armor, even if it shattered or reflected the bullets, and it wouldn't get stronger… --Tardis (talk) 15:02, 2 July 2008 (UTC)[reply]
I suppose if you had a billion rounds of machine-gun bullets, you could eventually wear a hole in the armor, but that's just getting a bit silly, the occupants of the tank would starve to death first. There may be a few weak spots, though, like the exhaust port, which might be more vulnerable to small arms fire. StuRat (talk) 15:38, 2 July 2008 (UTC)[reply]
I suppose water dripping on the tank would eventually wear through the armor, too. Friday (talk) 17:28, 2 July 2008 (UTC)[reply]
Removable armor plates are common. If the tank is disabled and just sitting there, you could rush up and start removing armor. Eventually, the guys inside will try to come out to stop you and it will be man vs. man instead of man vs. tank. -- kainaw 17:33, 2 July 2008 (UTC)[reply]

(EC)

What about MacGyver solutions? Take the gunpowder out, make a kind of bomb, could at least blow the tracks off.

Could destroy the main gun in a similar way. With no movement or firepower you could burn the people out...--Shniken1 (talk) 17:37, 2 July 2008 (UTC)[reply]

I don't know if the Barrett M82 would be useless against the tracks of a tank. It have caliber .50 and the latter are the Achilles' heel of the tank. Just give it a try on a tank that you don't need anymore. Mr.K. (talk) 09:54, 3 July 2008 (UTC)[reply]

The infantry could lay siege to the tank until the crew starve or surrender. Daniel (‽) 19:39, 3 July 2008 (UTC)[reply]

Evolution - Woman as Well?

Today I was speaking to a Jehovah's Witness that came by my door. We got into talking about everyone's favorite topic, evolution. He brought up an objection to the theory that I had not heard before. I don't remember exactly what it was, but it was along the lines of "even if a man evolved into a human, what are the chances that a woman evolved too, at the same place, and the same time?". I'm sure there is an easy rebuttal to this, but I was wondering if this objection has a name and how I can rebut it in layman's terms. (In a somewhat related query, does anyone know where one can find very old (circa 1894) copies of the Watchtower magazine?) Thanks! Abeg92contribs 03:56, 2 July 2008 (UTC)[reply]

Let me see if I understand the argument first. The idea is that if a male evolves from Species A to Species B, then there has to be a female that goes from Species A to Species B as well, otherwise there would be no-one to breed with? I can see what's wrong with the argument, but I'm finding it hard to put into words, except to point out that (a) evolution isn't sudden jumps, it's a gradual process (at least gradual in the sense that new species don't suddenly crop up in a single generation), (b) speciation occurs when an entire hereditary line becomes so genetically different to another that they cannot reliably interbreed, so it's actually the combination of a man and a woman who are, by necessity, genetically different, passing a particular combination of those different genes onto their offspring that leads to the creation of a new species. I'd try to explain it in terms of the historical development of language, but presumably the Witnesses believe that all happened when God struck down the Tower of Babel. I will leave the understandable explanation to someone with more experience in the subject matter. Confusing Manifestation(Say hi!) 04:56, 2 July 2008 (UTC)[reply]
Put another way, your Jehovah's Witness friend is mistaken in thinking that the process of speciation is something that occurs in an individual. Populations become species, individuals do not. - Nunh-huh 05:06, 2 July 2008 (UTC)[reply]
In addition to speciation, the population genetics article may also be helpful. --arkuat (talk) 05:35, 2 July 2008 (UTC)[reply]
The entire mechanism of reproduction involves males and females sharing their genomes again and again. The amount of genome that differentiates males and females is tiny. Your friend seems to think that males and females are terribly genetically different. They aren't. And again, all members of the species are products of their mating. So the genome isn't going to get out of sync between the two of them. --98.217.8.46 (talk) 13:53, 2 July 2008 (UTC)[reply]
If you want a more technical answer, consider how traits are distributed upon reproduction. Say I have a hypothetical gene for red hair. (Hair is a bit more complicated than one Mendelian trait but for our example it is concrete.) It sits somewhere on one of my chromosomes. I mate with a woman who had a gene for brown hair. It sits on one of her chromosomes. Inside the sperm, egg, etc., half of my genes are randomly thrown together with half of her genes. It's entirely possible that all of our male or female children could have my gene expressed, or half with her gene expressed. The resulting children are a mix-and-match of our two gene pools. It isn't that the "female gene pool" is separate from the "male gene pool" and has to "evolve" at the same time. They'll evolve together, as long as none of the evolutionary changes are great enough to make evolving with another human impossible. The gene pool of course allows for a certain amount of flexibility (if it didn't, even simple reproduction with genetically different members of the same species would be impossible). Beneficial mutations have a certain chance of being passed down to children of either sex. Males and females are, again, genetically (and developmentally, up to a certain point) almost identical—they are both humans, and it is the firing off of different hormones at different times that differentiate the two physically, mentally, etc. --98.217.8.46 (talk) 14:09, 2 July 2008 (UTC)[reply]
The number of chromosome is different in man and chimp, lets asume that the last comon ancestor had the same number than one of the two species. So sometime ago a chromosome split or paired with another to get to the different number. This happened in one single individuum not to the whole group.What I know from genetics this makes interbreeding very complicated what it is seen in donkey and horse. Has this problem of genetics been solved yet?---Stone (talk) 14:25, 2 July 2008 (UTC)[reply]
There are lots of theories as to how this can work. Needless to say, yes, it seems pretty clear that it can work. Even in individual humans you can do weird things like have too many chromosomes (e.g. XYY syndrome, trisomy 21). It is not usually positive in such cases though, but the point is, it can happen—it doesn't mean automatic death of the organism or inability to reproduce. --98.217.8.46 (talk) 15:24, 2 July 2008 (UTC)[reply]
I haven't seen a definite resolution of this "problem", but from genetic sequencing of the human and chimp genomes, it's pretty certain that the Human chromosome 2 is equivalent to a fusion of two chimp chromosomes, including residual portions of the centromere and telomeres that are no longer functional. My best guess is that at some point in the past, a fusion event happened in a human ancestor, creating chromosome 2. At this point, all breeding partners to this individual still had the two separate chromosomes, but as the fused chromosome still had all the regulatory apparatus of both chromosomes, the cells were still able to line up and divide normally (although perhaps at reduced efficiency). Eventually for some reason (potentially unrelated to the chromosomal fusion) the fused chromosome came to be dominant (This can happen very rapidly - see founder effect). At that point the secondary centromere/etc. was superfluous, and could be lost through mutations, leaving us with a single chromosome. (See [8] for a related discussion - no endorcement implied, just the first decent result I got in searching) -- 128.104.112.147 (talk) 19:54, 2 July 2008 (UTC)[reply]
There seems to be a false assumption that males and females evolve completely independently of one another. This is untrue. For the most part, only the X and Y chromosomes evolve independently, in humans. I can try to make this into a reasonable objection to evolution:

"Since, in humans, the X chromosome determines (or at least triggers) female characteristics and the Y chromosome male characteristics, the two must evolve in parallel so that any change in females matches a corresponding change in males. For example, the tendency in females towards nurturing, homemaking, and gathering required a corresponding change in males toward hunting, to ensure that all the child-rearing and food provision needs of the group were met, utilizing a division of labor."

I suppose this is true, but, in cases where both sexes developed noncomplimentary traits, like everyone going hunting and leaving the children unattended, those traits would not be likely to be passed down. StuRat (talk) 15:30, 2 July 2008 (UTC)[reply]
Basically, he doesn't understand how evolution works, because he is making assumptions that aren't true. ScienceApe (talk) 04:55, 6 July 2008 (UTC)[reply]

He thinks there is a point where something is human, and the previous generation is not, but an individual does not change, it's an entire population. Eriorguez (talk) 22:30, 8 July 2008 (UTC)[reply]

What is the process of decompossition of the human body?

Do you need further information for that question? —Preceding unsigned comment added by Ros1701 (talkcontribs) 04:43, 2 July 2008 (UTC)[reply]

Have you read Decomposition#Human decomposition? Gwinva (talk) 05:04, 2 July 2008 (UTC)[reply]

Cavity in a Unifomly charged solid sphere..

consider a uniformly charged sphere with a cavity inside. the electric field inside the cavity is non zero and uniform... but if we draw a gaussian surface inside the cavity, the charge enclosed will be zero. so field zero on the surface... why the contradiction???? —Preceding unsigned comment added by 117.201.49.226 (talk) 07:46, 2 July 2008 (UTC)[reply]

The electric field is zero inside. I think you are confusing electric field and electrostatic potential. The latter is uniform inside and may or may not be zero, as you're free to define a point where it's zero. Icek (talk) 08:50, 2 July 2008 (UTC)[reply]
Maybe they were thinking of Shell_theory#Thick_shells - inside the 'solid' of this shell..
Shell_theory esp. Shell_theory#Inside_a_Shell may help here with the maths..
Shell theory explains the field (works for any inverse square relation ship) both inside and outside hollow spheres..
Note at the centre of the cavity the field is zero. I've no idea what you mean by "but if we draw a gaussian surface inside the cavity..." if you wan't the field then shell theory has a mathematical method of how to get it.

CLARIFY:Did you actually mean a solid sphere, or thick walled sphere with a hollow cavity inside or something else?87.102.86.73 (talk) 12:03, 2 July 2008 (UTC)[reply]

One answer "... inside the cavity, the charge enclosed will be zero. so field zero on the surface.." - the surface is not inside the cavity - it's on the boundary - which is different..87.102.86.73 (talk) 15:10, 2 July 2008 (UTC)[reply]

I hate to quibble, but nothing in the initial question stated that either: A) the cavity is spherical or B) the cavity is centered. Hence it could be inappropriate to jump straight to shell theory. Dragons flight (talk) 23:44, 2 July 2008 (UTC)[reply]

quote "consider a uniformly charged sphere..." - it's easily to miss..87.102.86.73 (talk) 12:05, 3 July 2008 (UTC)[reply]
That says nothing about the shape of the cavity within the sphere. Dragons flight (talk) 16:04, 3 July 2008 (UTC)[reply]
Actually, even if the field is neither centered nor spherical, it will still have zero electric field within, although yes, shell theory wouldn't apply. Someguy1221 (talk) 23:57, 2 July 2008 (UTC)[reply]
That's not true of a uniformly charged (in a volume sense) insolator. You are correct if you assume the sphere is a conductor, which also is not stated. Dragons flight (talk) 00:04, 3 July 2008 (UTC)[reply]

Can I get fully solved IIT-JEE question papers in Physics (mechanics only) of past 10-20 years?

I want to know whether I can get fully solved Physics question papers of past 10-20 years of the IIT-JEE examination from you. If you do this small favor for me,I'll be very very thankful to you.Please send them (if possible) as an attachment to your response mai

--Prateekgreat (talk) 10:22, 2 July 2008 (UTC)prateekgreat[reply]

I guess "Do your own homework" needs to be expanded to "Study for tests on your own." (I presume this person wants this as part of a review for his or her exam - not sure what that ITT-JEE exam is they're asking for.)
And...an attachment? I *really* hope the person means a link to a site with such, because I don't know how a person could even send it to the questioner personally.209.244.30.221 (talk) 21:54, 5 July 2008 (UTC)[reply]

What defines a yeast?

I posted this at Talk:Yeast but thought I should place it here too.

What defines a yeast? Do yeasts have some intrinsic trait that qualifies a particular species as being a "yeast" rather than just generically being a "fungus"? What is special about all (or most) of the species of yeast that qualifies them as yeasts? --Alecmconroy (talk) 15:07, 2 July 2008 (UTC)[reply]

According to yeast, yeast is unicellular. Otherwise, fungi are usually multicellular. I think that's probably the main distinction. — CycloneNimrod talk?contribs? 16:08, 2 July 2008 (UTC)[reply]

How fast does the "wind" go when we blow on something?

Just blew on a crwling bug (likely an ant) to get it off my book outside, and it made me wonder - how fast does that "gust" blow that we creat when we blow?

I'm thikning, just from experimenting, maybe 50 MPH or so at first, but then of course air resistance will slow it down a lot. Plus, the amoung of saliva probably affects things. Holding my hand about a foot away from me (like when blowing out candles on a birthday cake) it still feels gusty, but not too much; maybe 20 MPH or so?209.244.30.221 (talk) 17:35, 2 July 2008 (UTC)[reply]

Sneeze gives some speed estimates. I would expect a sneeze is about the upper limit for how fast we can blow. So, somewhere less than the speed of a sneeze, which is itself the subject of widely varying estimates. Friday (talk) 17:43, 2 July 2008 (UTC)[reply]

Tree identification

My parents had a tree in their back yard, which was about, if I recall, about 3-5 metres tall, and had purple waxy leaves. It shed them in the winter, and was able to survive winter temperatures of -50 degrees celsius. I think it may have lived for about 15 years, and produced wrinkled, cherry like fruit. —Preceding unsigned comment added by Sliver Slave (talkcontribs) 20:25, 2 July 2008 (UTC)[reply]

First guess, purple leaf plum? Seems to survive in the cold, not sure about -50C though. --Wirbelwindヴィルヴェルヴィント (talk) 21:45, 2 July 2008 (UTC)[reply]

Crude oil estimates

Where/how do they come up with estimates for how long our fuel is going to last us? I mean how can they know how much there is when its all underground? --212.120.246.239 (talk) 22:06, 2 July 2008 (UTC)[reply]

Directly, geologists can use techniques like seismic mapping to figure out how large an oil pocket is. Indirectly, they can measure how fast oil is being pumped from the ground -- once the rate slows down, about half the available oil has been pumped out. Interesting articles to read are Hydrocarbon exploration, Exploration geophysics, and Hubbert curve --Carnildo (talk) 22:40, 2 July 2008 (UTC)[reply]
And, of course, how long oil lasts also depends on our rate of consumption. If we conserve energy and switch to alternative energy sources, oil should last longer. StuRat (talk) 03:54, 3 July 2008 (UTC)[reply]
They also have to account for the oil we haven't discovered yet, which can be estimated based on how commonly we find it, and they have to pick an arbitrary point where oil becomes too expensive to extract, and thus not counting things like tar sands. They also have to pick a point for not having enough fuel, as their will always be some left, but the speed at which we extract it will approach zero. — DanielLC 15:51, 3 July 2008 (UTC)[reply]
The traditional point of "too expensive to extract" is when the energy return on energy investment drops below 1: it takes more energy to extract the oil than can be gained by burning the oil. --Carnildo (talk) 21:46, 3 July 2008 (UTC)[reply]
Tar sands are counted as crude oil reserves, that's how Canada leapt at a stroke to become the nation with second-highest crude reserves. And in the case of the tar sands, the resource is at the surface, so it's relatively easy to estimate just how much is there. Franamax (talk) 01:04, 4 July 2008 (UTC)[reply]

Drafting

When driving on the interstate highways, if one wants to "draft" behind a fast moving eighteen wheeler, how close does he have to follow? WSC —Preceding unsigned comment added by 75.85.203.191 (talk) 22:29, 2 July 2008 (UTC)[reply]

Mythbusters did an episode on this. The benefit depends on how close you are: at a safe following distance, you get about a 5%-10% increase in fuel economy; if you're following at ten feet, your fuel economy doubles. --Carnildo (talk) 22:43, 2 July 2008 (UTC)[reply]
A word of caution: A truck driver once told me he really worries when someone drafts behind him. For one thing, you're driving in his blind spot when you're close enough to draft, and furthermore, truck tires sometimes throw their treads (you can see truck tire treads occasionally lying on the shoulders of freeways). These tire treads are heavy steel-belted rubber things that can fly right through your windshield. =Axlq 23:07, 2 July 2008 (UTC)[reply]
I keep hearing "If you can't see my mirrors, I can't see you", warnings about following too closely. I would expect that the truck driver would be more concerned about what's in front of him and to either side, since that is what controls his ability to maneuver. The obvious danger of drafting or tailgating is that the trucker slams his breaks on (presumably for good reason) and you don't, but that danger exists whether the trucker can see you or not.
A related question I've always wondered about: Has anyone ever studied the effect of drafting of the truck? I would think that the car would either (1) cause an increased low pressure behind the truck, thus causing additional drag and effectively stealing the trucks energy, or (2) the car would act as a tapered tail to the truck, thus decreasing the drag in the truck. -- Tcncv (talk) 23:33, 2 July 2008 (UTC)[reply]
A word of caution: with the Mythbusters episode in question, even the farthest distance tested was too close to constitute a safe highway-speed following distance.
As for the question of what drafting does to the truck's fuel economy -- interesting. You can't get something for nothing, but I don't know if car+truck would closely approximate a closed system for those purposes or not. — Lomn 00:11, 3 July 2008 (UTC)[reply]
The truck is already losing energy through air resistance - the car may well just be using that energy, so the truck doesn't lose any more. --Tango (talk) 00:15, 3 July 2008 (UTC)[reply]
At 55 mph, they got an 11% reduction in fuel usage at 100 ft and 39% reduction at 10 ft (Mythbusters' wiki). Whether or not you personally consider 100 ft to be safe may be something of a matter of opinion. It's not that uncommon on highways where I live to see cars following big rigs within that distance (which is about 6 car lengths). A big rig isn't capable of stopping on a dime, so even if he did slam on the brakes (which let's be honest isn't that common on freeways), you'd have the benefit of his slower stopping time to react and stop or dodge. Back when I was growing up 100 ft was actually the recommended distance at 60 mph, but I understand that people now recommend 150 ft. So you be the judge of whether the small reduction at 100 ft is worth the (in my opinion) small risk. Dragons flight (talk) 00:26, 3 July 2008 (UTC)[reply]
Given the huge mass the 18-wheeler carries, is it possible that even if you include reaction time that you would be able to brake safely in a normal vehicle?--droptone (talk) 12:14, 3 July 2008 (UTC)[reply]
By this reasoning a bicycle would out-brake a car, which is clearly not the case. Heavier vehicles have more powerful brakes, plus more wheels = more brakes to dissipate energy. Gandalf61 (talk) 16:17, 3 July 2008 (UTC)[reply]
Empirical evidence says otherwise. I haven't compared my 15-0 stopping distance to that of a bicycle, but I have compared 60-0 stopping distances with a semi (I was passing it when a car ahead of both of us lost control), and my car stopped well before the semi did. --Carnildo (talk) 21:54, 3 July 2008 (UTC)[reply]
The limiting factor in braking a bicycle is not the brakes. It's the tires, geometry, and weight distribution. The question is not whether you can slow down the wheels, but whether you can keep control of the bike when you do. Optimal braking is supposed to be around 70-30 or 75-25, weighting the front brake more, because the front tire presses into the pavement during braking and is therefore less prone to skidding. But hit the front brake too hard and you go over the handlebars. --Trovatore (talk) 22:15, 3 July 2008 (UTC)[reply]
Oh, that said--I think the drafting thing is seriously a bad idea. At highway speeds you should not cruise closer than 3 seconds behind the vehicle ahead of you. At 60 mph that's 264 feet. Of course you will have to get closer than that at times; you just shouldn't stay there. And if you don't stay there, you're not going to get much benefit in fuel economy. --Trovatore (talk) 22:47, 3 July 2008 (UTC)[reply]

Hi. What species of biting fly lives in southern Ontario, has a mildly painful bite, thrives near midday in June-July, and leaves a barely noticeable reddish bump about 1mm wide? I'm not asking for a diagnosis, just a vague species identification. The article doesn't seem to help, as it wasn't a mosquito, Tetse flies live in Africa, horse and deer flies are yellowish in colour, and black flies should be far more painful? I think it was grey-black in colour, had rufous-brown eyes, and was less than 1cm long. Its bite was that of a mild sting. It might sound like a blackfly, but aren't their bites more severe and larger? Thanks. ~AH1(TCU) 22:35, 2 July 2008 (UTC)[reply]

Around the Great Lakes, I've been bitten by many basic black biting flies. It is a mild sting - no worse than a mosquito bite. I've been bitten by deer flies also - it is severe. One caused my whole ear to swell up. So, I see no reason for it to be anything more than a basic biting fly. -- kainaw 01:03, 3 July 2008 (UTC)[reply]

Why are Galaxies disc shaped?

Just curious why some galaxies are disc shaped and not 3D collections of stars (like how I imagine Nebulae are). --70.167.58.6 (talk) 23:30, 2 July 2008 (UTC)[reply]

Rotation - and nebulae are gas, not stars. Rmhermen (talk) 00:03, 3 July 2008 (UTC)[reply]
He wasn't saying nebulae were composed of stars actually. He was just referring to the shape. ScienceApe (talk) 18:50, 3 July 2008 (UTC)[reply]
Nebulae often contain stars as the condensing of the gas is what causes the formation of stars. Its why we can usually see them, the gas is illuminated by the stars in there. EagleFalconn (talk) 00:14, 3 July 2008 (UTC)[reply]
Galaxy formation and evolution and spiral galaxy are probably good places to start. As Rmhermen says, it's all to do with the rotation - when you rotate something, it generally flattens out. Just watch someone showing off while making pizza - rather than roll out the dough, they spin it round and it flattens out nicely. The reason for the difference in shape between spiral galaxies and nebulae, I would guess, is the speed of rotation (possibly, relative to their overall mass - it might be angular momentum that's actually important) and the length of time they've existed. It's going to vary from galaxy to galaxy and from nebula to nebula, though. Plenty of galaxies aren't disc shaped (see elliptical galaxy and irregular galaxy), and there may well be some disc shaped nebulae (I honestly don't know!). I hope that helps a little. --Tango (talk) 00:13, 3 July 2008 (UTC)[reply]


July 3

House water pressure from flow rate?

Can I determine the pressure of my water lines just from the flow rate and the physical dimensions of the tubing? Thanks.

I measured 2.1 GPM through a 1/2" copper tube.

--jcmaco (talk) 01:35, 3 July 2008 (UTC)[reply]

You may be able to get an approximation (e.g. water will flow through a hole at a rate determined by pressure based on its pressure coefficient) but in the case of water lines, you don't have any way to account for friction or restrictions upstream. When the water isn't flowing, your water pressure is independent of restrictions or friction that would affect the flow.
I recently measured my pressure after installing a reverse osmosis water filter in my kitchen. The manufacturer provided a table showing how much water would be drainable out of the storage tank for various water pressures. I drained 1.8 gallons out of a 4 gallon tank (meaning that my water pressure was sufficient to push 1.8 gallons into the tank), which equated to a fairly low pressure of 40 psi. =Axlq 05:43, 3 July 2008 (UTC)[reply]
Not unless you could work out the resistance of the pipes to water flow. A much easier way to get your pressure would be to use a manometer.if you cant get hold of a mercury manometer, one could be made with a long length of garden hose with a transparent bit at the open end. Having connected the other end to your tap, you climb up a ladder holding the transparent end. When the water stops flowing out the end, the pressures are equal and you measure the height of the tube above the ground. Since pressure is height times density, you know the pressure on your system. —Preceding unsigned comment added by 79.76.137.118 (talk) 02:45, 6 July 2008 (UTC)[reply]

Tell diffrence

If man and woman are berried in same grave how can u tell the diffrence after 1000 years/ —Preceding unsigned comment added by 79.76.174.168 (talk) 01:51, 3 July 2008 (UTC)[reply]

Human skeleton#Sex-based differences. PrimeHunter (talk) 02:02, 3 July 2008 (UTC)[reply]
I'd expect the boys to be blue-berried. :-) StuRat (talk) 03:38, 3 July 2008 (UTC)[reply]
But blueberries are "false berries". Should that be taken into account? The questioner really should have specified which kinds of berries are to be used. I feel that strawberries would do nothing more than stain the bones a little red. Red currants are highly acidic. Use enough of them and you might make skeletal identification difficult. -- kainaw 05:13, 6 July 2008 (UTC)[reply]

Skin color of ancient Egyptians

So I've come across the debate of whether ancient Egyptians were black, white or something in-between. It seems that neither side has conclusive proof. My question is whether there are any accounts by travelers about the skin color of the country they had visited? If there are, then why is the debate still going on? —Preceding unsigned comment added by 24.7.54.224 (talk) 02:50, 3 July 2008 (UTC)[reply]

Mostly "white", or what essentially is the ethnic group we now call Arabs, I believe. Ancient Egypt consisted of Lower Egypt, in the North, and Upper Egypt, in the South, which were later united. South of that was Nubia and the kingdom of Kush, which, I believe were populated, then, as now, by "blacks". Since there was some interchange of populations, I'd expect some "blacks" to have been in ancient Egypt, especially Upper Egypt, as well. StuRat (talk) 03:27, 3 July 2008 (UTC)[reply]
As for evidence, there were many surviving color paintings in Egyptian tombs, so there's no need to resort to 2nd or third-hand accounts. Here's one that shows a nice racial blend: [9]. Forgive me if I break into a round of Ebony and Ivory while viewing it. :-) StuRat (talk) 03:31, 3 July 2008 (UTC)[reply]
Some people get confused about this because Cleopatra was white, but she had only four great-grandparents and all of them were of European descent. In general, you'd expect the skin shade of ancient Egyptian people to be about the same as that of peoples from around the world at the same latitudes, which would probably count as "somewhere in-between" on OP's proposed scale. --arkuat (talk) 03:44, 3 July 2008 (UTC)[reply]

It boggles my mind how anyone can have only four great-grandparents. Having eight should be more typical, no? Thank you for your replies, StuRat and Arkuat, but if what you said is correct, I don't understand why the debate still goes on. —Preceding unsigned comment added by 24.7.54.224 (talk) 05:02, 3 July 2008 (UTC)[reply]

Why the debate still goes on is perhaps a question better taken to the Humanities desk, but I think it has to do with the intellectual history of the African diaspora. Personally, I feel that it is important to recognize that people from all over Africa contributed to ancient Egyptian civilization, mostly for the simple reason that this is true.
As for having only four great-grandparents, that comes easy if your family culture encourages first-cousin marriage, or aunts marrying nephews, or uncles marrying nieces, or, as was expected of Egyptian royal families off and on since time immemorial, sisters marrying brothers. --arkuat (talk) 05:39, 3 July 2008 (UTC)[reply]
See pedigree collapse. Rmhermen (talk) 13:54, 3 July 2008 (UTC)[reply]
Pedigree of Don Carlos, illustrating pedigree collapse: grandparents (4) in yellow, great-grandparents (4) in blue
You may also be interested in Serious inbreeding among European Royals, which features several with four great-grandparents. - Nunh-huh 03:08, 4 July 2008 (UTC)[reply]
Good thing they didn't have Christmas then, talk about getting ripped off for presents! Franamax (talk) 19:55, 3 July 2008 (UTC)[reply]
We have an article on this debate: Race of ancient Egyptians. Rmhermen (talk) 13:54, 3 July 2008 (UTC)[reply]
"How" is quite simple: her grandparents on her father's side were siblings, and her grandfather on her mother's side was the brother of her paternal grandparents. The family tree at Cleopatra#Ancestry shows a remarkable degree of inbreeding. --Carnildo (talk) 22:02, 3 July 2008 (UTC)[reply]

The current majority in Egypt are arabs I believe, but they weren't the original inhabitants of Egypt when talking about ancient Egypt. I believe the copts are survivors of the original inhabitants of ancient Egypt. ScienceApe (talk) 17:26, 3 July 2008 (UTC)[reply]

That's true. At that time the Arabs were still mostly on the peninsula. The simplest way to think about it in todays terms is imagine Egypt and Sudan as one, sometimes two, kingdoms that keep taking eachother over. Depending on which group is dominating when, sometimes the pharaoh looks roughly like a black Sudanese and sometimes he looks like a Coptic Egyptian. There's obviously been a lot of intermixing with the Arabs but if you go back that's always been common by the sea, what with the Helens and Philistines and whatnot. -LambaJan (talk) 16:55, 6 July 2008 (UTC)[reply]
The Arab expansion has been mostly cultural, not genetic. It brought a new religion and often a new class of rulers, but not a massive population shift. The vast majority of Egyptian population remained of ancient stock, and has by now thoroughly absorbed many population groups, including Greeks, Jews, Romans, Arabs and Turks. One point of confusion may be that for most mediterranean people, skin colour is very much influenced by lifestyle. If they stay indoors, they remain pasty white. If they spend a lot of time in the sun, they get a very dark tan. I'm not officially mediterranean (well, who knows what exactly my ancestors did for fun ;-), but I have a set of passport photos taken 4 weeks apart. On the one I look like an unhealthy Englishman, on the other like a Southern Indian. The difference is two weeks of beach volleyball in June in Greece. --Stephan Schulz (talk) 17:52, 6 July 2008 (UTC)[reply]

hypoglycemia vs hypertriglycerides

what is the relationship of sugar to triglycerides? is it normal to have hypoglycemia with high triglycerides? —Preceding unsigned comment added by Docbenjie (talkcontribs) 05:54, 3 July 2008 (UTC)[reply]

Well, firstly hypoglycaemia is not normal. Secondly, hypoglycaemia (lack of glucose) is not affected by the level of triglycerides (which are fats). It's true that fat can be used as an energy source when glucose has been used up but that does not affect hypoglycaemia itself. That's just my knowledge though, if this affects you medically you should see a doctor. — CycloneNimrod talk?contribs? 11:53, 3 July 2008 (UTC)[reply]
Hyperglycaemia (high sugar), seen in poor diabetic control, is associated with higher triglyceride levels. Not sure direct link, rather via common factor of hyperinsulinaemia (raised insulin levels in those who are insulin resistant due to obesity)... for a quick synopsis, see the PubMed abstract of: Reaven GM, Javorski WC, Reaven E (1975). "Diabetic hypertriglyceridemia". Am. J. Med. Sci. 269 (3): 382–9. PMID 168773.{{cite journal}}: CS1 maint: multiple names: authors list (link)David Ruben Talk 13:28, 3 July 2008 (UTC)[reply]
Cyclone: Hypoglycemia is not normal, but it's also not really abnormal, if that makes sense. Your body is constantly operating to maintain homeostasis, but sometimes one parameter or other shifts too much and your blood sugar drops. It's abnormal the same way dehydration is abnormal: it's an abnormal state of the body that can occur transiently to otherwise healthy people. Sometimes folks who exercise to hard without carb loading or people who eat an especailly carb heavy meal end up experiencing attacks of hypoglycemia. See Hypoglycemia#Causes and Reactive hypoglycemia --Shaggorama (talk) 07:13, 4 July 2008 (UTC)[reply]

Why don't spaceships turn into blackholes

According to Einstein, as a spaceship goes faster and faster towards the speed of light, the mass of the spaceship gets larger and larger while the lenght of the spaceship gets shorter and shorter.

So why don't spaceships turn into blackholes? 122.107.135.140 (talk) 12:17, 3 July 2008 (UTC)[reply]

Interesting question... I've never thought about it. I think it's just a matter of perspective, though - the spaceship gets denser from the point of view of a "stationary" observer, but it's density remains constant from its own point of view, and that's the reference frame you need to work in to tell if it's going to collapse into a black hole. --Tango (talk) 12:23, 3 July 2008 (UTC)[reply]
The question assumes you could accelerate the spaceship to a meaningful fraction of the spead of light, but as its mass increases, so does the force needed to accelerate further. Using the Lorentz transformation a ship at 99% the spead of light has a Mass 7 times that before and a length one seventh. Tango is correct that onboard every seems as normal, but to an outside observer indeed the density (M/V) would seem to be 7 / (1/7) = 49 times as great, which for any reasonable size ship is not going to have any real gravitational effect. 99.99% of c gets us to outside observer measured denisty x5000 and 99.9999%c to 500,000 density; so still not even in our thought experiments getting anywhere close to a density that might cause us problems. Schwarzschild radius indicates one would need to compress the Earth's 6,371km radius into just 9mm radius to become a black hole; that's a compression of over 700million !
See also Gravitational singularity#Curvature, about our assumptions about measurments of weight and distance need to be rethought.
Finally see Micro black hole#Creation of micro black holes for the effort to smash just a few atoms together (accelerator of 1000 light years) so I think we're safe from any fast spaceship ! David Ruben Talk 13:19, 3 July 2008 (UTC)[reply]
There is no physical limit on how close to the speed of light you can get. It may not be physically possible even under the most extreme assumptions to do it, but we can still discuss what would happen (as you can probably tell, I'm a mathematician, not a physicist!). So, if I got in a spaceship and travelled past the Earth at such an enormous that I observed the Earth's mass and dimensions to be such that it ought to collapse into a black hole, what would I see? The Earth obviously doesn't actually collapse, from the point of view of someone on Earth, but what would I see? I've been thinking about this, and I just don't know. --Tango (talk) 13:40, 4 July 2008 (UTC)[reply]
What, you think you might see the Earth collapse, even though it doesn't? No, that's not going to happen. Relativity isn't that relative. Gravitation is controlled by the stress-energy tensor, which is a frame-invariant quantity.
That's not to say that velocity is never relevant at all. The invariant mass of a complex system involving multiple bodies that are not at rest with respect to one another does depend on their relative velocities. I can imagine something like, say, a system of two sets of rods approaching each other very fast, both in checkerboard patterns so that they slide past each other. The invariant mass of this system would depend on the speed at which that happened, and maybe if you made that speed high enough you could get the system to undergo gravitational collapse in principle. --Trovatore (talk) 17:32, 5 July 2008 (UTC)[reply]
Oh, or here's another way of putting it: Suppose you fly your ship fast enough, and close enough to the Earth, that the Schwarzchild radius for the invariant mass of the combined system (Earth+spaceship) envelops both the Earth and your ship. Then, perhaps, the combined system collapses, both you and the Earth, and both you and observers on Earth agree this happens, though they may not agree on why. There are some possible gotchas here, not sure whether the Schwarzchild solution is still a solution for a system this asymmetrical, but it's at least sort of plausible. But note that the rest mass of your ship is a key ingredient. If your ship is just a massless, energyless, metaphysical point of view flying past, there is no speed at all that would cause the Earth to collapse, either as it observes it or as you do. --Trovatore (talk) 18:05, 5 July 2008 (UTC)[reply]

The rest mass doesn't change. Only the relative mass changes. The matter that makes up the ship doesn't increase, nor does it become more dense. In order for a black hole to form, you need to mash a large mass into a very dense form. Like collapsing Mount Everest into something smaller than a human cell. ScienceApe (talk) 17:20, 3 July 2008 (UTC)[reply]

Also, the situation described sounds an awful lot like a situation where you'd need quantum gravity. -RunningOnBrains 22:59, 3 July 2008 (UTC)[reply]
Our article on mass in general relativity answers this question. It says "Can an object move so fast that it turns into a black hole? No. An object that is not a black hole in its rest frame will not be a black hole in any other frame ...". There is also a link to this page in the Usenet Physics FAQ. Gandalf61 (talk) 14:55, 4 July 2008 (UTC)[reply]
Thanks Gandalf61 for pointing to that answer. I also not the later Q&A re heated atoms possibly having more mass because thay have kinetic energy when heated. David Ruben Talk 20:53, 4 July 2008 (UTC)[reply]

Wheat allergies

Hi. I'm curious about wheat allergies as someone I know has one. Could it be that such a person would have a worse reaction from eating a "whole wheat" product (say pasta) than a regular one? --Dweller (talk) 13:42, 3 July 2008 (UTC)[reply]

Wikipedia cannot, does not, and will not provide medical advice. That said, you may want to read our Wheat allergy article, a more medical website, or consult a physician. Plasticup T/C 13:57, 3 July 2008 (UTC)[reply]
I'm not looking for medical advice, nor do I need to consult a physician as there's nothing wrong with me. I'm just curious. But thanks for the articles - I'll read them. --Dweller (talk) 14:24, 3 July 2008 (UTC)[reply]
The first of those articles does mention whole wheat, but the reference comes in an incomprehensible paragraph. The second article does not mention whole wheat. --Dweller (talk) 14:29, 3 July 2008 (UTC)[reply]
Everything I look at merely lists foods to be avoided, with refined wheat and whole-wheat considered to be the same. It could be that the difference in allergic reaction, if any, is not significant enough to be mentioned. Fribbler (talk) 14:37, 3 July 2008 (UTC)[reply]
Thanks. --Dweller (talk) 14:47, 3 July 2008 (UTC)[reply]
Based on reading the wheat allergy article, it looks to me like, because there are many allergenic components in wheat, you get all of those components with whole wheat and less of them with processed wheat. The processing may remove some of the minor allergens (but seems to preserve the main one, gluten). A hypersensitive person could conceivably have a worse reaction if more allergens are present. ~Amatulić (talk) 17:43, 3 July 2008 (UTC)[reply]
Also, remember that allergies are funny things. People with a severe allergy can have a bad reaction to tiny amounts of allergen and past reactions are not necessarily indicative of future reactions. So the distinction between whole wheat and refined wheat products may be entirely irrelevant. 86.141.89.124 (talk) 23:39, 4 July 2008 (UTC)[reply]

botanical name

What is the botanical name of the tree that is supposed to go to sleep when the sun sets and rise up again with the sun? Its leaves droop down in the night and freshen up in the morning.

Shyam59.92.72.54 (talk) 14:47, 3 July 2008 (UTC)shyam[reply]

With heliotropism, flowers and leaves track the sun as it moves through the sky. At night they either default back to facing east or just some random orientation. -- MacAddct  1984 (talk &#149; contribs) 16:31, 3 July 2008 (UTC)[reply]
There are several types of trees that do this. Albizia julibrissin is the one that came to mind first.--Eriastrum (talk) 23:19, 3 July 2008 (UTC)[reply]
I like the ones you touch and they fold, like Mimosa pudica. Then there's one with a folding name: Prayer plant. But they're not trees. Julia Rossi (talk) 09:56, 4 July 2008 (UTC)[reply]

Species with long luteal phase

I am interested in which species have a long luteal phase of the estrus cycle. So far I only know that dogs and elephants fall in that category. Any other species? Thanks in advance. Arisa. —Preceding unsigned comment added by 131.211.166.194 (talk) 15:05, 3 July 2008 (UTC)[reply]

After googling a bit, I gotta admit this information is difficult to find. I did confirm that elephants have the longest cycle. I thought maybe other large mammals like whales may also qualify, but I couldn't confirm that. ~Amatulić (talk) 21:59, 3 July 2008 (UTC)[reply]

Material acting like Maxwell's demon?

Hi all,

Is there anything theoretically impossible about a material, say some kind of not-yet-invented cloth made out of nanotubes, which only lets fast-moving molecules through? How about a material which only lets molecules through in one direction? I can't see anything wrong with the idea except that it may never be invented. But wouldn't such a material essentially act as Maxwell's demon and break the Second Law of Thermodynamics (allowing us to create order out of disorder)?

All the arguments against the demon seem to involve the energy that he must exert separating the molecules, or the energy that he adds observing them, or the lack of infinite memory to know where they all are (I've never really understood that one). This material, however, wouldn't need to "know" which molecules are fast any more than a coffee filter "knows" that water can pass through and larger particles can't. And likewise, like a filter, it wouldn't need to do any work to let the fast molecules through.

So is there anything theoretically wrong with this idea?

Thanks! Sam 16:51, 3 July 2008 (UTC)

I don't really see anything theoretically wrong, per se, with a membrane that would only allow things moving quickly to pass through. As I think about it more, theres plenty of those in existence...pretty much everything fits the criteria. Its just a matter of how you define fast. The penetration of cosmic rays through buildings and such is a good example. Unidirectional motion is also possible, but usually in cases like veins where you've got some kind of pressure being exerted and valves used to control flow.
A common error made when people look at the Second Law of Thermodynamics is they forget the caveat to that rule. You cannot create order out of disorder without adding energy. There is energy in the membrane. If it stops slow moving particles, then it is doing some kind of work from some energy source (mechanical, chemical or otherwise) to stop them. If we were to use a piece of paper as the membrane, its still doing that same work on a fast moving particle, but the fast moving particle has so much energy that it keeps going anyway and breaks the bonds or whatever. This seems a little convoluted, but I hope it helps. Lemme know if I should try again...EagleFalconn (talk) 17:14, 3 July 2008 (UTC)[reply]
This is an interesting question. Although I am certainly not the scientist that Maxwell was, nor do I claim to be anything close, I will give it a try. First, this is a thought experiment for a reason. We have several ideal parts of this system: the box, the particles, the wall, the evil-looking green demon, and an ideal separation from the rest of the universe. It is hard to imagine anything close to this level of ideality in a real world system. With that said, the second law of thermodynamics is built to work for ideal systems, too.
One comparison you draw is the difference between a coffee filter and a fast-molecule filter. This is a broken simile I think. A coffee filter distinguishes water from grounds based on size. Energy is a totally different beast. One of the facets of the uncertainty principle is that momentum and position cannot be measured simultaneously. This is because in the act of measuring, the particle is perturbed, changing the other property. In the same way, if a barrier were created such that only particles of a certain energy could pass through, it would strip the particles of that energy as it passed through the barrier, eliminating any gain in entropy. I say this carefully though, because if the particles being discussed were electrons, by using a tunnel barrier you could take advantage of the quantum tunnelling effect and allow particles of certain energy to pass through without such a loss in energy. The risk here is that tunnelling is itself a probabilistic process. ANY electron could conceivably tunnel without losing energy. Higher energy ones just have a larger probability of making the leap so to speak.
What all of these theories lack, however, is an impetus for separation. In your coffee filter example, water and beans are on the same side of the filter and gravity pushes the water through. Now imagine trying to operate the same system in a zero-g environment. That wouldn't really work. Sure all the grounds would stay on one side, but the water would be everywhere. In the same way, unless some gradient were created to move the particles in the right direction, equilibrium would still exist. If you started with all particles on one side and allowed them to move to the other via any of the previously mentioned methods without field, the particles could just as easily move back again, given a high energy. What you are talking about is some sort of thermodynamic diode or valve that requires no potential.
My last comment is this: great thinking. Diodes exist, spin valves exist, many things for separating particles of different properties exist. All need energy. Minimizing that energy is key. It is possible that Maxwell's demon may never exist, but something close to it (operating on the energy of room temperature, for example) would not violate the second law, and may well be very useful. Gjmulhol (talk) 17:18, 3 July 2008 (UTC)[reply]
Thanks for the two comments. In response to the argument about the "impetus for separation," I'm not sure that this is needed. (1) If we have a zero-g box with coffee and water on one side of a coffee filter, eventually there will be a mix on one side and pure water on the other, so some separation has occurred (even if not all the way), and (2) If it is a one-way filter (like I mentioned above), eventually each water molecule will have randomly gone to the other side and not been able to get back, thus completing the separation.
I think you're both probably right about the fact that it may be impossible to separate fast- and slow-moving molecules without slowing down the fast-moving ones -- making them rip through the filter, for instance. (I don't think, however, that a barrier has to do work on a slow-moving molecule to make it bounce off -- the molecule's energy would be maintained).
Does the energy that it seems that a molecule loses as it goes through the barrier have to be so much that the fast molecules that pass through it a turned into "slow" molecules? By which I mean, would it be theoretically possible for a barrier to stop all molecules with V < x, and when fast molecules (V > x) break through, their speed is still greater than x? If so, we're still creating a potential difference between the two sides of our box that could be used to generate work, right?
Thanks, Sam 17:45, 3 July 2008 (UTC)
Well, I think we need to distinguish between molecules and electrons here. I am not an expert in physical chemistry. I do have some training in solid state physics. When an electron impinges on a surface, it can do one of 3 things:
  • bounce back
  • tunnel through
  • go over
By go over, I mean have enough energy that it is able to go through the barrier. If the barrier is an energetic one, which I think is what we are talking about here, it will sacrifice the energy used to overcome that barrier and fall to the lowest state available. If you were to erect a barrier of a specific height x (to use you example from before), then the only reason that the particle can pass is that it has enough energy to give. It is like buying a ticket, if you don't have the money, you don't get in. If you do have the money, you get it, but you leave the money.
I still stick with my p-n junction argument that I posted a few minutes ago farther down. I think that is the closest thing of a separating barrier. A solar cell is based on the exact principle described here, but you need that pesky addition of sun to provide some nominal amount of energy to cause current flow.
I honestly have no idea whether there is some corollary to molecular movement as I have described electron movement. Gjmulhol (talk) 23:28, 3 July 2008 (UTC)[reply]

Slightly different question, brought to mind from the comments above: Is a one-way filter alone theoretically plausible? If so, would this be a great way to produce usable energy out of un-ordered states? Say I strap a one-way filter to one side of a box, so that air can only go in. The pressure in the box can only increase. If I punch a hole in the other side and add a turbine, my one-way filter will create a stream of air, which could be used to generate electricity, right? Any theoretical objections to that one? Thanks again! — Sam 17:51, 3 July 2008 (UTC)

Well, other than being a perpetual motion machine... (The act of ordering states from unordered states requires energy. Pure and simple. Any instance when you find yourself suddenly getting lots of energy out without putting any energy in is a sure sign that something has gone wrong in your calculations.) --98.217.8.46 (talk) 18:26, 3 July 2008 (UTC)[reply]
The input energy in his example is from the random movement of particles in the air. I don't see any immediate flaws. Oh, and I like to think of your theoretical material as fish trap. Plasticup T/C 19:26, 3 July 2008 (UTC)[reply]

Probably the best example of something that seems to violate the second law is an optical isolator. But even a hypothetical perfect optical isolator doesn't do it, not that I really understand the explanation [10] Someguy1221 (talk) 19:37, 3 July 2008 (UTC)[reply]

One example of something similar is a solar cell. Although it does absorb energy, based on the p-n junction's built in potential, electrons flow one way and holes the other. This creates a current. It is a one-way system. Even when no light is on it, there is some small generated 'dark' current from heat effects. Like I said though, it is not a closed system and does require some sort of energy. Gjmulhol (talk) 23:16, 3 July 2008 (UTC)[reply]
(ec)Scale matters for this question. Newer research points out that the Second Law of Thermodynamics does not apply to nanoscale systems.[11] I read a few weeks ago[citation needed] that one of the most promising future ideas in nanotech is harnesing Brownian motion to power molecular machinery.-RunningOnBrains 23:18, 3 July 2008 (UTC)[reply]
I would be very interested to see the full text of that article. Much of the BBC coverage seems simplistic for a such bold claims. Do you have the DOI, author name, or date of the article? I know from personal experience that forces at the nanoscale level are much different than at even the micro level. For example, capillary action and surface tension are the dominant forces in any system with nanoscale features. I would guess (and remember I haven't yet read the article, so I don't know the details) that the Second Law of Thermodynamics is not being violated. There must be energy coming from somewhere (the shaking of the canister? heat from the room? potential energy of the charge on the bead? again, I don't even know what they measured). Do the researchers make such claims of violating the second law explicitly? If so, this paper really should have been in Nature or Science rather than PRL ;).
Much Brownian motion comes from heat. Motion cannot exist without energy. By harvesting the energy from Brownian motion, we will simply be absorbing heat. This could be done from phonons or diffusion current. Gjmulhol (talk) 23:36, 3 July 2008 (UTC)[reply]
The second law of thermodynamics has been put through every thought experiment anyone can think of. Have some faith in these scientists. Don't ask if your idea will work; ask why it won't. As for why it won't:
I don't see how a material that only lets fast-moving particles through can be used to break the second law of thermodynamics. Maxwell's demon requires it to only allow fast particles one way and only allow slow the other. I don't know if it's possible.
The one-way filter is impossible. In addition to breaking the second law, it wouldn't be time reversible. After writing the next part, I noticed it could be time reversible.
I don't know much about the optical isolator, but I'm going to guess that it either can't be made reflective (so entropy is gained from light turning to heat). It's time-reversible because switching the direction time runs would switch the direction of the magnetic field, causing the isolator to face the other direction, and if it can absorb light, it can also spontaneously emit it.
This part's not really related, but it's a pet peve of mine: Gjmulhol, the uncertainty principle is because of inaccuracies in measuring. It's possible to make multiple particles with the same quantum properties, and to measure different properties in different ones, thus bypassing the observer effect.
98.217.8.46 and Plasticup, this isn't about creating energy; it's about destroying entropy.
RunningOnBrains, they don't go into much detail so I'm going to say that it's A: talking about the fact that at a small scale, entropy has some chance of decreasing, but won't consistently, or B: wrong. Also, take a look at the Brownian ratchet, which is based on building up small scale chance decreases in entropy, and still doesn't work. — DanielLC 22:46, 4 July 2008 (UTC)[reply]

Shooting down the barrel of a tank

Kinda inspired by a previous question. Lets say you have a high caliber machine gun, and you shoot down the barrel of the main gun of a tank with full automatic fire. What would happen? Would the cannon be damaged? Could it still fire as if nothing happened? Would the bullets cause a "clog" eventually? ScienceApe (talk) 17:16, 3 July 2008 (UTC)[reply]

That depends on a couple of things: (a) your aim, and (b) the cannon bore size. You could probably fairly easily shoot a 50-caliber machine gun down the barrel of a 155mm howitzer cannon, without the bullets touching the cannon barrel.
Even if the bullets did touch the side of the barrel, they impact at a grazing angle and will likely not cause any more damage than a cannon projectile being shot through it.
If a cannon barrel is clogged, you wouldn't want to fire the cannon, or you risk rupturing the barrel. ~Amatulić (talk) 17:30, 3 July 2008 (UTC)[reply]
Well basically what I'm asking is if you took a 50 cal and fired down the barrel of the cannon of a tank. Would it damage it, and/or would it clog it? ScienceApe (talk) 18:48, 3 July 2008 (UTC)[reply]
If there was a shell loaded, you might be able to detonate the shell (cover your face if you're looking down the barrel). Otherwise, the bullets would strike the breech - likely they wouldn't damage it all that much, unless you hit the firing mechanism, but you could stick enough lead onto the breech to make it unusable to seat the shell. And yes, if you keep firing an infinite number of bullets, eventually you'll clog the barrel with hot lead. Franamax (talk) 19:39, 3 July 2008 (UTC)[reply]
Ah... I assumed your original question was about firing a gun through a tank barrel from the inside of the tank. You're asking about firing a gun into the muzzle? That could damage the breech if the cannon isn't loaded, not only in the way Franamax describes, but also in that some cannons use a laser to detonate the propellant, and you could damage the laser window. If the gun is loaded, you might hit the fuzing mechanism of the round, or even clog the barrel, which would get cleared out when the cannon is fired, possible with damage to the barrel. However, I doubt you would do serious damage to the barrel by firing into it due to the grazing angle the bullets make with the inner wall of the barrel. ~Amatulić (talk) 21:53, 3 July 2008 (UTC)[reply]
If you simply roll a rock down the barrel or pack some mud in it and induce the tank crew to fire through the barrel, that should cause the gun to explode , possibly killing the crew. This recommendation is from a friend who says this was a tactic used against German tanks in WW2 when it was infantry against armor with no antitank weapons. A tank without infantry support is not all that secure against infantry attack. Edison (talk) 14:11, 4 July 2008 (UTC)[reply]
Imagine that - Sarge hands you a rock and says "go fight that tank" :) Franamax (talk) 23:05, 5 July 2008 (UTC)[reply]

Strange skull

Hi guys, I was wondering if somebody could identify the animal this skull belongs (belonged :p) to:
It was found on the beach somewhere in northern Crete. It is around 5-6 cm in length.
http://img20.imageshack.us/img20/2782/scull1ad2.jpg
http://img20.imageshack.us/img20/4995/scull2gc4.jpg
http://img503.imageshack.us/img503/6638/scull3ft8.jpg
Thanx a lot!
PervyPirate (talk) 17:25, 3 July 2008 (UTC)[reply]

At first I thought it might be some sort of bird, but it looks like there are teeth in the front. A big rat, maybe? It doesn't look like what I'd expect for a fish. Great pictures, though. If someone can identify it, and Wikipedia has an article on it, please put that picture in the article. ~Amatulić (talk) 17:35, 3 July 2008 (UTC)[reply]
Was I the only person who saw those and immediately thought 'dragon'? :) At a guess, I'd say some sort of predatory eel... --Kurt Shaped Box (talk) 17:46, 3 July 2008 (UTC)[reply]
That was my first guess as well, since moray eels are quite common in that part of Greece, but a quick google image search proved me wrong... PervyPirate (talk) 17:54, 3 July 2008 (UTC)[reply]
Are you sure it's a skull? It looks to me like a pelvic bone. Compare the third image above with this of a seagulls pelvis: [12]. Fribbler (talk) 17:57, 3 July 2008 (UTC)[reply]
Wow! I would never have guessed! Thank you Fribbler (and everybody else)! However, since it's not a skull after all, I doubt the pics will be of any use in an article. PervyPirate (talk) 18:06, 3 July 2008 (UTC)[reply]
Why not? Bird anatomy in the skeletal section, or pelvis (currently an overly human-centric article) could benefit from one of those nice, clear pics! Fribbler (talk) 18:11, 3 July 2008 (UTC)[reply]
Fair enough. I will ask permission from the friend who made the pictures, upload them properly, and update the talk pages of the articles you mentioned. PervyPirate (talk) 22:12, 3 July 2008 (UTC)[reply]
Pelvis? Whoa. How utterly unexpected. Shades of three blind men and an elephant? ;) --Kurt Shaped Box (talk) 18:37, 3 July 2008 (UTC)[reply]
Or at least a few men who haven't spent much time around comparative anatomy courses... skulls are pretty distinctive, and even though I arrived to this question pretty late in the game it was clear that those pictures weren't of a skull. --98.217.8.46 (talk) 19:18, 5 July 2008 (UTC)[reply]
Great pictures by the way, you've got some skill, like that as a desktop background...(got any more?)87.102.86.73 (talk) 19:49, 3 July 2008 (UTC)[reply]
To be honest, these are not really my pics. But I'll pass the good words to the friend who made them. Cheers! PervyPirate (talk) 22:12, 3 July 2008 (UTC)[reply]

basic difference between stroke and depression?

Okay, first, let me assure you I'm not looking for a diagnosis. Rather, I have a question more about the basic differences in these two.

It seems in depression, some of the basic themes apply as to those in strokes. A person may have difficulty speaking, moving, etc.; becuase strokes apparently don't have to affect only one side of the body. And, that in severe enough depression, close to a nervous breakdown, such things can, in fact, stop, just like in a stroke.

Is it common for people to mistake the two? Or, to miss a stroke in a person who has severe depression? Becasue, if the person can't bring themselves to move, and suddenly can't move, how do you know the difference?

Also, has anyone ever tried stroke recovery methods for combatting depression? My guess is it wouldn't work, but a lot of those symptoms are seeming the same right now.

I'm sure things that confuse the common person (like myself) are very clear to someone who has spent 4 years in medical school, another 4 in residency :-)209.244.187.155 (talk) 18:25, 3 July 2008 (UTC)[reply]

They are totally different. Tennis elbow and amputation might both leave you with a sore arm, but they are not related in any way. A stroke is a physical disease and depression is a psychological one. Plasticup T/C 18:47, 3 July 2008 (UTC)[reply]
Depression does not cause paralysis! Jdrewitt (talk) 19:50, 3 July 2008 (UTC)[reply]
Perhaps you're confusing depression with a total nervous breakdown? They can prevent the body from functioning, but only because the central nervous system is so ovreloaded. It's not a clinical term per se, but a pshycological condition, in general, is a function of chemical imbalance, I believe, not like a stroke, where it is somply a blood clot blocking something. Also, during a nervous breakdown, cells don't die. (I'm putting it in very simple terms, but seening as you could be confused between physical and psychological conditions themselves, so I felt that might be helpful.)
I'm not sure that stroke therapy could help with a nervous breakdown, becasue the object of stroke therapy is to teach the brain to function again. (Again, making it very simple.) My guess is that it could help to some small degree, if the person views him or herself incapable of simple tasks (for instance, they had stopped speaking), and they need to be encouraged to do them...but that would only be part of the treatment.
I know this kind of goes off the OT, but...I'm struggling to understand the nature of the question, and I felt I had to try *something*. Somebody or his brother (talk) 01:08, 4 July 2008 (UTC)[reply]
One general way to think of it is that a stroke is neurological -- lack of oxygen to the brain -- whereas depression is psychological (very broad generalization that I realize treats chemical imbalances too lightly). As they are so different in nature, the treatment of one doesn't really apply to the other. One important difference between the two is that a stroke (also reffered to as a brain attack) is an emergency condition. If chronic depression is left untreated, over time the person may lose control of their lives and potentially kill themselves. If a stroke is left untreated for a matter of hours, the individual may suffer permanent nerve damage and die within days. --Shaggorama (talk) 06:59, 4 July 2008 (UTC)[reply]
There are several reasons why these two conditions should not be confused by a competent medical professional. Firstly the onset of the two conditions will be different. In the case of a stroke caused by a thrombus (as opposed to a cerebral bleed) there will be a fairly quick onset, varying from instantaneous to tens of minutes. Immobility caused by depression is likely to take weeks or months to develop, a gradual process that should be apparent to the person's family or friends. The presentation of the two conditions are quite different. Following a stroke the person will have variable paralysis and impairment of conciousness depending on the severity of the thrombus. There will be specific neurological signs present in a person who has suffered a stroke that will be absent in a person who has impaired movements caused by deep depression. In a person with incapacitating depression all their concious mental and physical bodily processes are inhibited but in a person who has suffered a stroke there will be some variability in the movements of the body, and their thought processes (those still working) will react at normal speed. The main difference will however show itself in the onset of the condition. It is very unlikely that 'stroke therapy' will assist someone suffering from a deep depression - or vice versa. Richard Avery (talk) 07:07, 4 July 2008 (UTC)[reply]
While trying to avoid giving actual medical advice, let me also add that a person who has suffered a stroke can lapse into depression - i.e.: person who suffers is too weak to get out of bed easily, or paralyzed for a short time with a transient ischemic attack, winds up becoming depressed about this and may begin to get more depressed, but the depression does not relate directly from the death of brain cells, but rather from the notion that said person is not able to function at their previous level. (So, to the untrained eye, it may *look* like they're related in that instance, when they're not. I shan't delve too much further, though, because it is possible that there is a medical concern on the Op'ers part, and that this is where the confusion lay.Somebody or his brother (talk) 22:02, 5 July 2008 (UTC)[reply]

Gnat ID

I'm trying to identify a Gnat that has a black abdomen which is clearly separated from the thorax (although this could be an illusion produced by a black intrail or its contents and a clear thorax) like that of an ant and extremely large red eyes and relatively short wings which is about a mm long.

The remarkable thing about this Gnat is its ability to hover and to precisely adjust its location to maintain a distance of about 3 to 6 inches from any object which comes close and to relocate itself to its original position when the object moves away in sync with the objects motion.

What species might this Gnat be? -- adaptron (talk) 19:57, 3 July 2008 (UTC)[reply]

Could be a fruitfly. Take a look at this [13].--Eriastrum (talk) 23:14, 3 July 2008 (UTC)[reply]
Yes, very close for the eyes, thorax also is orange but only a transparent tint. The legs are also clear. Wings, however, are much shorter. The hovering behavior is so controlled and precise its scary - as if being operated remotely in real time by a much bigger brain. :-) -- adaptron (talk) 23:31, 3 July 2008 (UTC)[reply]
...as a matter of fact here it is... Drosophila melanogaster. -- adaptron (talk) 23:33, 3 July 2008 (UTC)[reply]

July 4

Trivalve moluscs

Do these actually exist? It's just that I seem to remember reading something about them years and years ago somewhere...[dubiousdiscuss] --Kurt Shaped Box (talk) 00:30, 4 July 2008 (UTC)[reply]

I see trivalves mentioned fairly often in fantasy fiction and science fiction. There's also this story about Constantine Samuel Rafinesque concerning a discovery of a trivalve mollusk, which was a joke played on him by John James Audubon. There's also a page from Google books about the incident. ~Amatulić (talk) 00:52, 4 July 2008 (UTC)[reply]

Is tungsten ferromagnetic?

I checked the article, and all it said was no data... ScienceApe (talk) 06:12, 4 July 2008 (UTC)[reply]

I am pretty sure the answer is no. Graeme Bartlett (talk) 06:48, 4 July 2008 (UTC)[reply]
No, it isn't. Though some tungsten-containing compounds are. 131.111.228.15 (talk) 07:44, 4 July 2008 (UTC)[reply]

Well I know they use tungsten projectiles in railguns, so if it isn't ferrous, then how do they propel it? ScienceApe (talk) 16:32, 4 July 2008 (UTC)[reply]

The projectile (and rails) of a railgun only need to be good conductors; no ferromagnetism required. Algebraist 16:40, 4 July 2008 (UTC)[reply]
Seems like it. Thanks. ScienceApe (talk) 17:52, 5 July 2008 (UTC)[reply]

where do herring gulls and black back gulls sleep at night?

Does anyone know what sort of places they usually sleep in? They seem to all leave the town and the local landfill area at night and all go off in the same direction. I'd like to be able to drive out and see them sleeping one night. —Preceding unsigned comment added by 84.67.233.220 (talk) 07:29, 4 July 2008 (UTC)[reply]

I believe Gulls are naturally cliff dwellers, I imagine they would roost on cliff faces if they're near the sea. For urban gulls, buildings would present a similiar landscape. —Preceding unsigned comment added by 62.25.96.244 (talk) 09:42, 4 July 2008 (UTC)[reply]

Just to note that I am not a Gull and am not called Cliff. --Dweller (talk) 15:05, 4 July 2008 (UTC)[reply]

Battery-powered microphone amplifier

Can someone suggest a battery-powered circuit for amplifying the output of an electret mike to drive a headphone (the type commonly used for portable media players)? The circuit should draw very little power, require no more than 2 AA batteries, and be buildable using easy-to-find parts. —Preceding unsigned comment added by 71.175.20.73 (talk) 11:45, 4 July 2008 (UTC)[reply]

You might look at [14], they are a huge producer of ICs and sell all kinds of audio amps. You would probably be best to purchase from [15]. Depending on your requirements, this could be a good starting point for selecting them [16]. You should know the impedance of your headphones. The data sheets of all these audio amplifiers should have sample schematics of simple circuits that you can start from. Depending on your input, you might need a preamp. It is also important to note that if it is a condenser mic, you will need to bias the microphone as well. Gjmulhol (talk) 12:13, 4 July 2008 (UTC)[reply]
On a side note, be careful about grounding. Audio signals are very sensitive to bouncing ground planes. Gjmulhol (talk) 12:17, 4 July 2008 (UTC)[reply]
"Grounding" is important, as is shielding of microphone leads, to prevent hum from influence of the powerline frequency electricity, but certainly no earth-ground connection would be needed. Edison (talk) 14:04, 4 July 2008 (UTC)[reply]

Termite control in Scarborough, Ontario, Canada

Dear Wikipedians:

Does anyone know some termite control shops that are located in Scarborough, Ontario, Canada?

Thanks.

74.12.39.232 (talk) 15:31, 4 July 2008 (UTC)[reply]

Hi. Have you tried yellowpages.ca? Thanks. ~AH1(TCU) 15:41, 4 July 2008 (UTC)[reply]

Overlapping tidal radii

Hi. On an issue of SkyNews magazine, I read that stars can have large zones where an object will orbit it, 1.5 - 15 light-years in radius. It said that the Sun's zone, called the tidal radius, is 3.5 light-years. It also said that they can overlap. This made me wonder.

The nearest star system, Alpha Centauri/Rigil Kentarus/Toliman, is 4.3 light-years away from us. If the sun's tidal radius is 3.5 light-years, I'd expect Alpha Centauri's radius to be a bit larger as it has higher total mass, say 4.0 light-years. This leaves 3.2 light-years of overlapping tidal space at the horizontal plane diametre. Well, is it plausable that AlphaCen has an oort cloud of comets, just like the sun?

I would like to know (and this is not homework, and I don't know if I'm able to calculate this):

  • The volume of overlapping space, in cubic light-years;
  • The number of Oort cloud comets estimated to be orbiting the Sun in that space;
  • The approximate percent of the Sun's tidal sphere surface that is within the overlapping zone;

Or, is the tidal zone not a sphere, but an elliptoid, just like the Heliophere? Oh no!!! Is the spherical model a good approximation? Or is it possible to calculate the same parameters for an elliptoidical model? Would we know in which direction the tidal elliptoid extends farthest and its shape, if it is indeed an elliptoid? Why is the Heliosphere elliptoidical, does its direction change over time, what direction does it point in, does it have any effects on comets just entering the Heliosphere, and could its shape be caused by Nemisis?

Consider the known non-returning comets, with eccentricities of ≥1. Do we know any which have the open end of the parabola(/hyperbola?) pointing towards the AlphaCen direction? Might they be from/going towards the AlphaCen system? Might some be coming back? Are there any other star systems which have tidal radii overlapping ours? Three or more overlapping together? Could this explain some Oort cloud collisions that send the comets plunging towards the sun?

How will AlphaCen's negative radial velocity change this zone over time? What about Gliese...(something, forgot its number, in Ophiuchus) and Barnard's Star as they approach our Sun? Is there an article concerning the subject of orbital tidal radii (and I'm not referring to the Roche limit, which "orbital radius" redirects to)? Might this explain some extinctions?

Using current extimates, about how many comets are sent towards our sun due to the overlapping tidal zone from AlphaCen per each specified specific period of time (eg. per year, per century, per millenium, and how many actually arrive)? Or is it not possible to answer some of these questions due to limited knowledge data? Thanks. ~AH1(TCU) 16:17, 4 July 2008 (UTC)[reply]

I've never heard of "tidal radius" before, and our article on it redirects to Roche limit, which has nothing to do with what you're talking about. The closest term to what you describe that I've heard is Hill sphere, but that's for one object orbiting another, rather than two separate stars. So, I'm not really sure what a tidal radius is meant to be (I can't see what your description has to do with tidal forces, for a start!). The maximum distance at which you can have a stable orbit around a star is going to depend on the other stars around it, and there won't be any overlap (they might go right up to eachother, though, I'm not sure if there would be a region of instability inbetween or not). It's certainly not going to be a sphere. An orbit around the sun in a plane perpendicular to the direction of Alpha Centauri could be larger than the distance to Alpha Centauri, I suppose, but one in the same plane as Alpha Centauri would be unstable if it got too close to the star. --Tango (talk) 16:47, 4 July 2008 (UTC)[reply]
A (crude) diagram used to vaguely illustrate the concept of stellar "tidal radii". ~AH1(TCU) 01:54, 5 July 2008 (UTC)[reply]
Hi. What I (and the magazine) mean by tidal radius might have nothing to do with tidal forces. It states that close orbits around a star can be stable, but the tidal radius, according to the magazine, is, beyond which, an object will no longer be within the gravitational reaches of the star.
Within the "tidal radius", objects like planets, comets, and other bodies are close enough to the star to be able to orbit it. Beyond that distance, the gravitaional forces of the rest of the galaxy cause any object to be lost to the star's grip. It also says that wandering comets might enter the Sun's tidal radius, perhaps from another star's orbital field, and be captured by the sun's gravity and form an orbit.
I see how a star's gravitational field can be affected by other stars, but perhaps it is referring to the zone in which a minor body (and perhaps another star) can be affected by/orbit the star. It also refers it to as the "tidal limit". It also says that some stars' tidal limits can overlap. The tidal limits may, indeed, go right up to another star, but I don't think it does in the case of Sol/AlphaCen.
The magazine goes on to approximate the tidal zone as a sphere, saying Altair's stretches 30 deg across the sky. It says that some comets may have entered the tidal limits of other stars or were flung out to empty space to become "vagabonds", and that those might enter our Sun's tidal radius and possibly may have been seen from Earth, but that is speculation.
Also, some orbits of comets are not a complete ellipse, but have two parallel or angled ends, forming a parabola. My question is, might some of those parabolic comets' orbits have come from or are heading towards the orbital boundaries of other stars? What about AlphaCen in particular?
The estimate of 3.5 light-years for our Sun's tidal boundaries would suggest that an orbit leaving this zone, probably already parabolic as opposed to elliptical, would be lost from the Sun's gravitational force and become an interstellar "vagabond". If it enters AlphaCen's zone of gravitational attraction but have already exited ours, might it form a new orbit, perhaps still parabolic due to its great distance of entry, around AlphaCen?
The distances in space are great, but if say AlphaCen had an Oort cloud similar to ours, and our Oort clouds overlapped, could a select few comets be influenced by other comets' orbits and gravitational attraction, so that their own orbits end up destabilised and plunge towards one of the stars?
If a comet was heading towards or around the AlphaCen system, could Proxima, itself more massive than Jupiter and (currently) closer to us than AlphaCen itself, act like Jupiter does in our solar system, flinging comets' orbits towards AlphaCen or our own Sun? If a comet approaches AlphaCen, could interactions between stars A and B cause a star-grazing comet to act differently than one near the Sun, and have its orbit drasticly changed (if it survives)?
Also, and this is speculation: if comets were theorised to have been one of the harbingers of life on Earth, might some of those same comets, containing similar organic compounds, have survived an interstellar journey and struck a planet in another star system, producing life there? Could the same have happened to Earth, from other star systems? Thanks. ~AH1(TCU) 01:16, 5 July 2008 (UTC)[reply]
I can't see anything to stop comets from our Oort cloud being perturbed and ending up orbiting another star, but I expect it would be very unlikely due to the distances involves (the perturbation would have to be very precise, or it would miss the other star). If this kind of thing did happen, it could possibly transfer life between stars, this is called exogenesis. However, I really don't see how the regions in which you can have stable (closed) orbits around two stars could possibly overlap - how would an object in that overlap "know" which star to orbit? If you had an object orbiting one star and an object orbiting the other such that their orbits intersect at one point, their apoapsis (further distance from the star they are orbiting), directly between the two stars. Then, at that intersection point, both objects would be travelling in the same direction (perpendicular to both stars), although possibly at different speeds. One object would then need to fall towards one star and the other towards the other star, but why would they go in different directions if they started out going in the same direction? The objects could "choose" to fall towards either star, so the orbits would be unstable. --Tango (talk) 01:32, 5 July 2008 (UTC)[reply]

convert -

A hole guage of 10-32 -> diameter(inches) —Preceding unsigned comment added by 75.60.90.55 (talk) 19:50, 4 July 2008 (UTC)[reply]

I'm assuming that 10-32 is refering to a screw size. Take a look at [17] for dimensional information. -- Tcncv (talk) 20:02, 4 July 2008 (UTC)[reply]
Or here: Unified Thread Standard. --Heron (talk) 21:02, 4 July 2008 (UTC)[reply]

Simple mechanics.

Let's say there's a stick lying on the ground and I kick it. Well depending on where I hit it and at what angle, the stick will gain some linear momentum and some angular momentum. If I always kick it staight on (i.e. perpendicular to the stick), then only the position matters. So my question is, how can I determine how much angular and linear momentum the stick will possess as a function of the kick's position? Now I gave this question a shot, and my guess is that because the stick has a bit of thickness, the angle between the force and the radius of rotation changes depending on rotation, and that product of the sin of the angle and the force would give the angular acceleration and so on. Is this right? A second and somewhat related question, why does the stick always spin around the center? —Preceding unsigned comment added by 65.92.4.238 (talk) 22:43, 4 July 2008

This is actually a bit difficult to work out (and easily confuses me still)
Basically it depends on amongst other things, the way the weight in distributed in the stick, I'll assume that the stick is straight and has equal thickness - so that the weight is evenly distributed along the length of the stick.
You can work this out using newtonian mechanics, you need to know about conservation of momentum, conservation of angular momentum (see also torque)
Let's say the stick is L long and you kick it at distance d from the middle (ie d<L/2)
(You also need to be able to calculate the Moment of inertia of the stick by integration, this is in fact mL2/12 (from List_of_moments_of_inertia) IF you want to calculate the exact numbers)
Suppose your kick at distance d gives an impulse 'S' to the stick, this impulse can be split into rotational and translational (straight line) momentums..
Assuming that the stick rotates about the centre for simplicity.
The rotational momentum can be viewed as one end of the stick moving forwards at speed V2 and one end moving backwards at speed V2, the translational motion can be viewed as the stick moving at speed V1 from the middle (combining all these gives the overall motion)
The linear motion is therefor mV1 (m is the mass of the stick) (ie the momentum)
The speed of rotation about the centre is given by V2=L/2 x A (A is the angular speed of rotation in radians per second.
The rotational torque is given by speed of angular rotation x angular moment of inertia = A x m x L2 /12
since A = 2 x V2/L the torque = V2 x m x L /6
If you supplied an impulse of 'S' to the stick at distance d the torque about the centre was S x d
So S x d = V2 x m x L /6
and S = mV1
So m x V1 = V2 x m x L /6d
ie rearanging gives V1/V2 = L/6d
This means the ratio of the angular speed and linear speed can be calculated and it depends on how far from the centre it was kicked.
Additional work gives a general formula for collisions of two objects using similar methods.
(Apologies for any mistakes I've made whihc I'm sure other will point out..) The method is similar to what I've tried to describe above.87.102.86.73 (talk) 01:46, 5 July 2008 (UTC)[reply]

Your second and related question is one I find difficult to explain.. BUT if you consider the stick mentioned above, and consider it rotating about somewhere other than the centre of mass - you'll find that the stick would be violating the 'laws of conservation of momentum' since it's centre of mass would be moving about.. there for its momentum would be changing over time without an external force (ie newtons law that a body moves in a straight line unless acted upon by another force - in this case the body would be effectively 'spiraling about a point') - in general if you try to make a stick do this it will tend to rotate about it's centre and any extra speed will simply turn into linear momentum. Sorry if I haven't explained that very well.87.102.86.73 (talk) 01:52, 5 July 2008 (UTC)[reply]

Bounce

Why does a ball bounce when it hits the floor? —Preceding unsigned comment added by 65.92.4.238 (talk) 22:44, 4 July 2008 (UTC)[reply]

When it impacts the floor, it deforms by flattening at the bottom, this absorbs the kinetic energy of its motion and converts it into elastic potential energy. That elasticity then causes the deformation to undo, the ball returns the being spherical and is propelled upwards. Does that help? --Tango (talk) 23:27, 4 July 2008 (UTC)[reply]
Why does a steel ball bounce higher than a tennis ball when the tennis ball deforms more then? --antilivedT | C | G 01:15, 5 July 2008 (UTC)[reply]
Because more energy is lost to heat and sound when the tennis ball deforms and reforms than when the steel ball does. I'm not really sure why... --Tango (talk) 01:34, 5 July 2008 (UTC)[reply]
I have never seen a steel ball bounce higher than a tennis ball... Plasticup T/C 03:47, 5 July 2008 (UTC)[reply]
You haven't bounced it on the right surface. Try bouncing it off of another piece of steel. Someguy1221 (talk) 09:38, 5 July 2008 (UTC)[reply]
There's an excellent demonstration of this at the Exploratorium. There, the steel ball bounces off a very large piece of steel and each bounce of the ball is nearly as high as the previous bounce (so very little energy is being lost on each bounce).
Atlant (talk) 20:55, 5 July 2008 (UTC)[reply]
Or off another steel ball, as in the Newton's cradle. There's a good reason why they make those things out of steel balls and not tennis balls - steel is more elastic than a rubber bladder. --Heron (talk) 13:41, 5 July 2008 (UTC)[reply]
Okay cool. So theoretically, an object which is perfectly rigid would not bounce, right? —Preceding unsigned comment added by 65.92.4.238 (talk) 04:09, 5 July 2008 (UTC)[reply]
That's an excellent question. I think it depends on how you take limits. A "perfectly rigid" body should have an infinite elastic modulus, because it never has any strain, regardless of its stress. So one logical way to end up with a "perfectly rigid" body is to take an elastic body and let its elastic modulus go to infinity. When this body is at rest on the ground, the gravitational potential energy it started out with is completely in the form of elastic energy, which I believe is . If the elastic modulus continues to increase, the stress must also increase to keep the elastic energy the same, so our "perfectly rigid" body would end up having infinite stress (in order to have a finite elastic energy with zero strain). If you accept this, then the body should bounce perfectly elastically (i.e. it should return to the same height it was dropped from and never lose any energy), because inelasticities are caused by strain. —Keenan Pepper 05:23, 5 July 2008 (UTC)[reply]
An alternative way of looking at this would be to say that a perfectly rigid body has no way of absorbing/losing the energy on impact (since it cannot deform)- therefor it would bounce perfectly.87.102.86.73 (talk) 12:07, 5 July 2008 (UTC)[reply]
For illustration, see the ball bouncer demonstration in the references of the Liquidmetal article, showing how steel balls bounce against three different materials: steel, titanium, and a metallic-glass compound called LiquidMetal. =Axlq 18:11, 5 July 2008 (UTC)[reply]

From Collision: "Collisions can be elastic, meaning they conserve energy and momentum, inelastic, meaning they conserve momentum but not energy, or totally inelastic (or plastic), meaning they conserve momentum and the two objects stick together." I'm not sure what perfectly rigid would mean, but the type of ball that doesn't bounce at all is called totally inelastic (the energy is absorbed by making the floor move down). --Tango (talk) 12:27, 5 July 2008 (UTC)[reply]

Well not really. The ball could have collided with the earth in a perfectly elastic manner, but the mass of the earth made its movement imperceptible. —Preceding unsigned comment added by 65.92.4.238 (talk) 17:02, 5 July 2008 (UTC)[reply]
It's not actually possible for something perfectly rigid to exist. Because of that, it's impossible to say what would happen if it did. If you were to magically fix the atoms in position with relation to each other, the electron orbitals in the atoms would distort as they get closer, so the atoms themselves wouldn't be perfectly rigid. If you were to throw two such bodies against each other, my best guess is that the enormous pressure would cause the two bodies to pass through each other somewhat, and the parts inside each other would be forced in every direction, canceling it out. The circle where the two surfaces overlap would still exert a huge pressure and force the two apart. More simply, they'd bounce off of each other perfectly elastically and extremely quickly, but still slowly enough that for a short time, they'd be partially inside each other. If only one body was "perfectly rigid" as I defined it, I have no idea what would happen. If you want to make the object more rigid by not having the electron orbits distort, their waveforms would take up the whole universe, and the Pauli exclusion principle would mean that no electron could have the same state as any in the rigid body, and they couldn't be the same as each other. Besides the ridiculously high energy states of the electrons this would require, it would also mean that said principle wouldn't be keeping the atoms from occupying the same space, so the rigid body probably wouldn't have any resistance from falling through the ground. Disclaimer: I'm not only not a physicist, but I only just finished high school, and the physics I learned there really doesn't apply here, so don't be surprised if I'm wrong. — DanielLC 16:29, 5 July 2008 (UTC)[reply]
Just because it can't exist, that doesn't mean we can't address the question. Usually when we deal with things like objects bouncing, we focus on the arangement of atoms indirectly by assigning certain constants, which were determined experentally, and don't take quantum mechanichs into account, just like we may, in classical mechanics, we cometimes treat an object as a single point, even though that would violate Heisenberg's uncertainty principle. So what my question is really asking is, using the laws of physics that we normally use to adress such phenomena as bouncing, how would a perfectly rigid body act? —Preceding unsigned comment added by 65.92.4.238 (talk) 17:13, 5 July 2008 (UTC)[reply]

Relativity and communication speed

If a spaceship traveling at near light speed were equipped with a radio-wave Internet connection, how would special relativity affect measurements of its bandwidth and latency? NeonMerlin 23:05, 4 July 2008 (UTC)[reply]

The latency depends on the distance. As the latency changes quickly, the time-out would need to be rather large or change with the predicted distance of the spaceship.
If the spaceship travels towards Earth, then the bandwidth is larger, if it travels away from Earth, the bandwidth is smaller (see Relativistic Doppler effect). This just means that the spaceship traveling towards Earth will receive the bits at a higher rate, and the one traveling away from Earth will receive the bits at a lower rate. Icek (talk) 07:48, 5 July 2008 (UTC)[reply]

July 5

Houdini exhumation

I followed the story of the proposal to exhume Houdini's remains. Some of his relatives wish to examine his remains to confirm whether or not he was the victim of arsenic poisoning, while others object to the exhumation. The latest information I can find dates from March, 2007. Joseph Tacopina represented the pro-exhumation faction then, but I would like to know who prevailed. Does anyone have current status on this case?Avid Djinn (talk) 00:08, 5 July 2008 (UTC)[reply]

Not exhumed as of 2008-03-04 according to:
Costella, Annmarie (2008-03-04). "Houdini exhumation to test for poison". NYDailyNews.com. Retrieved 2008-07-05. they have yet to file court papers to have Houdini's body exhumed.
-- Jeandré, 2008-07-05t18:26z

Future Indian Ocean Earthquake

Hi. Will there be another possible Indian Ocean Earthquake like the 2004 Indian Ocean Earthquake in the future? When will it happen again? What will be the possible magnitude of another future Indian Ocean earthquake? Sonic99 (talk) 00:52, 5 July 2008 (UTC)[reply]

It's not known. Earthquake prediction is poor. PrimeHunter (talk) 01:10, 5 July 2008 (UTC)[reply]
Hello again there Sonic, take a look here for some insight. ;-) -hydnjo talk 01:12, 5 July 2008 (UTC)[reply]
The answer to the first question is yes. There will be another very large earthquake on that fault eventually. Whether it will be tomorrow, next year, next decade, or 5000 years from now is largely impossible to predict. Dragons flight (talk) 01:45, 5 July 2008 (UTC)[reply]
According to Nature (journal), the recent earthquakes did not sufficiently release the accumulated fault strain and there is still the possibility of a magnitude 9.0 quake. The fault ruptures every 200-230 years and there is a 200-km zone that has not ruptured since 1797, so do the math.
Furthermore, it now appears that the active fault extends northward along the coast of Burma up to the coast of Bangladesh, and large quakes there would produce devastating tsunamis in the Bay of Bengal.
That said, the comments above about trying to predict exactly when an earthquake will strike are quite valid. (Although I did just read something interesting about using increased neutron flux for short-term prediction [18]). Franamax (talk) 22:59, 5 July 2008 (UTC)[reply]

Antarcticine

can anyone tell me more about this glycoprotein in terms of anti-aging? I have googled it but cannot find a suitable answerSwinstarr (talk) 02:46, 5 July 2008 (UTC)[reply]

There's a good bit of information in this article. Someguy1221 (talk) 09:34, 5 July 2008 (UTC)[reply]

Metrication in Greece

According to an 1866 report in the US House of Representatives, "Greece has introduced [the metric system] with some modifications." - I'm wondering what exactly these modifications are. It'd also be interesting in general to have more information about the history of metrication in places other than France and English-speaking countries. --Random832 (contribs) 07:33, 5 July 2008 (UTC)[reply]

For Greece, see Konstantinos Nikolantonakis' "Weights and measures: the Greek efforts to integrate the metric system" from section 5.1. For Italy, Spain, Portugal, and the Netherlands see the abstracts and PDFs at the 2nd International Conference of the European Society for the History of Science, Symposium R-8 pages. -- Jeandré, 2008-07-05t18:11z

Spin

Do electrons actually spin?? where does the intrinsic angular momentum value come from? —Preceding unsigned comment added by 116.68.73.39 (talk) 07:58, 5 July 2008 (UTC)[reply]

No one knows. Their electromagnetic properties (charge, magnetic dipole moment, conservation of angular momentum during particle interactions, etc.) exhibit behaviors similar to what a spinning macroscopic object with a non-zero charge distribution would produce, which is I believe why that characteristic is called "spin". --Prestidigitator (talk) 08:46, 5 July 2008 (UTC)[reply]
Overview of spin might be of interest. Jdrewitt (talk) 11:13, 5 July 2008 (UTC)[reply]
Some hold the view that the 'spin' is not in any way equivalent to a rotation, but is in fact comparable to a chirality of the electron - the two spins being equivalent to 'left' and 'right handed versions' of the particle.87.102.86.73 (talk) 12:35, 5 July 2008 (UTC)[reply]
Electrons have zero size but non-zero angular momentum. This is weird. Make your own mind up about what to call it. "Spin" seemed like a good word at the time. -- Tim Starling (talk) 13:35, 5 July 2008 (UTC)[reply]
Then how/why is the Classical_electron_radius defined? Jdrewitt (talk) 15:43, 5 July 2008 (UTC)[reply]
It answers how and why in that article you just linked to. — DanielLC 15:53, 5 July 2008 (UTC)[reply]
Well I wouldn't say it answers exactly how and why (at least why re isn't defined) but then that's quantum mechanics for you! Jdrewitt (talk) 15:58, 5 July 2008 (UTC)[reply]
Classical electron radius is a useful characteristic length for the extent of the electric field around an electron. However, the mass and charge of the electron is concentrated at a point at the centre. Models for the electron involving a distribution of charge have been rigorously disproven. Under classical electrostatic theory, this leads to the rather embarrassing result that an electron has infinite electric potential energy. Solving this problem was one of the early challenges for quantum electrodynamics. -- Tim Starling (talk) 00:17, 6 July 2008 (UTC)[reply]

Quantum numbers

I've learnt about quantum numbers describing electrons, but are there quantum numbers for other particles(fermions, in particular)? If they have spin, there must be other quantum numbers too. If so, how are they different from those of the electrons? —Preceding unsigned comment added by 116.68.73.39 (talk) 08:02, 5 July 2008 (UTC)[reply]

The quantum numbers you refer to come about from the wave equation for an electron "orbiting" the nucleus of an atom. There will be quantum numbers describing any particle in any kind of potential, but there may not be the same number of them and they may not take on the same values. Some quantum numbers (such as spin) do seem to be intrinsic to the particle though. The quantum number and spin (physics) articles may be a good place to start learning. --Prestidigitator (talk) 08:59, 5 July 2008 (UTC)[reply]

Can crouching, breathing deeply, rising quickly lead to fainting?

Is this an old wives' (or young high schoolers' :-) tale or not? Someone at school said if you bend your knees in a crouch, take bout 15-20 deep breaths, then try to stand up quickly, you pass out. Do you?

Either way, exactly what problems can this cause? I was tempted to try taking 20 deep breaths like that, then lay down from that position one night but even that I was too scared to do, just in case. Would that have caused a problem.

And, in case, there is anyone else reading this - please be like me, don't try it at home.209.244.187.155 (talk) 15:11, 5 July 2008 (UTC)[reply]

Finally fixed my edit - sorry for the apparent incompetence :-)
Anyway, I would think it might, by inhaling and exhaling deeply 10-20 times, you relax the muscles enough that they become too weak to supprt you right away, but aside from fainting I'm not sure there would be any other problem, unless you hit your head on something.
Still, I agree - NEVER do that.Somebody or his brother (talk) 15:14, 5 July 2008 (UTC)[reply]
Hi. Have you read our article on brownouts? Thanks. ~AH1(TCU) 16:10, 5 July 2008 (UTC)[reply]
I think this is similar to the choking game. --Mark PEA (talk) 17:26, 5 July 2008 (UTC)[reply]
This sounds like nothing to be worried about, but you might find Orthostatic hypotension and Hyperventilation interesting reads. Even if this did cause you to black out, I'd be more concerned about hitting your head when you collapsed than from hypoxic brain damage. --Shaggorama (talk) 00:40, 6 July 2008 (UTC)[reply]

I tried it inbetween the corner of two walls and a cushion. I crouched on the balls of my feet and bent to where I could keep my balance with my fingertips on the cushion and took 20 deep breaths, each of 2-1/2 to 3 seconds inhaling and 2-1/2 to 3 seconds exhaling. I felt a little faint at times breathing and when I was done I stood up quickly and felt very faint, particularly for the first couple of seconds after standing, but I didn't fall or feel in danger of falling. I came directly here to type this and I still don't feel entirely un-faint. I'm 25, male, and in average-good shape. -LambaJan (talk) 17:13, 6 July 2008 (UTC)[reply]

wow...uh, thanks for taking one for the team, I guess. Be safe. --Shaggorama (talk) 04:19, 7 July 2008 (UTC)[reply]
I doubt the breathing has all that much to do with it in terms of oxygen intake -- as Shaggorama said, it's probably just a form of head rush. Speaking as a non-doctor, I'd assume that the deep breathing serves to lower your heart rate and generally calms you down, which also lowers your blood pressure. When you get up quickly, hey, it's orthostatic hypotension time, baby! A comparable thing has happened to me once or twice. In one memorable instance, I was expecting a really important phone call and took a nap, and when the phone rang, I woke up and immediately dashed out of bed. I remember taking a couple of steps and heading for the phone, and then the next thing I knew, I came to on the floor feeling kind of nauseous and with the phone on my hand, evidently immediately after passing out, as the caller didn't even realize that anything out of the ordinary had happened. (I don't think I'm particularly susceptible to it, really, but I've certainly learned to not just spring on my feet as quickly as I can when I wake up.) -- Captain Disdain (talk) 12:02, 7 July 2008 (UTC)[reply]

electromagnetic force uses photons as a mediator

is this saying that when I play with magnets there are photons being exchanged?

How about when I play with built-up charges in a Van de Graaff generator? —Preceding unsigned comment added by 71.147.33.238 (talk) 16:15, 5 July 2008 (UTC)[reply]

From my shaky understanding:
  • When you play with magnets, you are moving a magnetic field, no photons are being exchanged per se. If your magnet causes electrons to move in a conductor, the moving electric charges will cause an electromagnetic field to emanate, this is propagated by photons.
  • Similarly, with a VdG generator, you are creating an electrostatic field, again no photons involved. When you ground the generator though, lets say through a spark gap, you can hear it on a radio - again, moving electric charges generate an electromagnetic field which propagates through space by means of photons which eventually strike the radio antenna and cause electric charges to move there.
Don't know if that makes it clearer and I welcome any other views on this. Franamax (talk) 20:12, 6 July 2008 (UTC)[reply]

Why do s-orbitals always take part in hybridistion?

In hybridisation of atomic orbitals, why do s-orbitals always take part?Ashudeep2singh (talk) 16:54, 5 July 2008 (UTC)[reply]

Hybridization leads to stabilization: if s overlaps p, the s electrons go down in energy (see LCAO). But more philosophically, why not? No seriously...the s orbital is spherically symmetric, so no matter what direction you look (i.e., the directional orbitals (p, d, etc) with which you might hybridize), there is electron-density there in the s. An s orbital can constructively overlap with p, d, etc because it is in the same place, and electron delocalization is a good thing. DMacks (talk) 18:52, 5 July 2008 (UTC)[reply]
To expand on that - when constructing a bonding orbital, it is impossible to ignore the s orbitals because they are spherically symmetrical.. you could imagine a p3 orbital instead of a sp3 orbital - but anything that bonds to that p3 orbital would probably have significnt orbital overlap with the relevent s orbital as well..
However in construction of say sp2 orbital you will note that the other orbital that remains is a single p orbital.. in this sense this gives an example of a hybridised set of orbitals in which one orbital does not contain s 87.102.86.73 (talk) 10:09, 6 July 2008 (UTC)[reply]

Platypus evolution

Regarding the platypus, according to this, http://en.wikipedia.org/wiki/Platypus#Evolution it seems the platypus contains some genes that are more similar to birds. This I found unusual because mammals evolved from Synapsids, and not birds, which evolved from true reptiles. So basically I'm asking how do you explain this similarity with birds, when mammals did not evolve from birds?

I would also like to know what kind of sex chromosomes reptiles have. The article said, "the sex chromosomes of the Platypus are more similar to the ZZ/ZW sex chromosomes found in birds.", well if amphibians and reptiles don't have those chromosomes, then that would raise a lot of questions about what platypus evolved from. The article in Nature said that the platypus shares two genes found previously only in birds, amphibians and fish. I'm not sure if they are referring to the sex chromosome though. But even if they are, they are insinuating that reptiles don't have those sex chromosomes. Well that's strange because birds evolved from true reptiles and not synapsids which probably did have those sex chromosomes. If reptiles don't have those chromosomes, well it raises a lot of questions. ScienceApe (talk) 17:51, 5 July 2008 (UTC)[reply]

Well, I think the point is that the platypus diverged from the rest of mammals very early on. My guess is that the genes in question that are no longer found in any other mammals probably have to do with the egg laying aspects, etc. To my mind the issue is not "what the platypus evolved from" but "what did the very early mammal genome look like?" It seems not unreasonable to me that it could contain all sorts of elements in it that were quickly filtered out (through a number of possible issues, like founder's effect) but the monotremes seem to have skipped out on the big filter. But this is just speculation. --98.217.8.46 (talk) 19:15, 5 July 2008 (UTC)[reply]
That doesn't really address the concerns I mentioned. In particular the sex chromosomes reptiles have. ScienceApe (talk) 23:41, 5 July 2008 (UTC)[reply]
I don't know much about sex chromosomes and evolutionary lineages but according to ZW sex-determination system, it's found in birds, fish and insects. It also mentions "Chromosomes in the ZW region in birds are autosomal in mammals, and vice-versa; therefore, it is theorized that the ZW and XY couples come from different chromosomes of the common ancestor. A paper published in 2004 (Frank Grützner et al, Nature; doi:10.1038/nature03021) suggests that the two systems may be related. According to the paper, platypuses have a ten-chromosome–based system, where the chromosomes form a multivalent chain in male meiosis, segregating into XXXXX-sperm and YYYYY-sperm, with XY-equivalent chromosomes at one end of this chain and the ZW-equivalent chromosomes at the other end." Sex-determination system is also helpful It sounds to me like platypuses have both ZW like and XY like sec chromosomes. My guess is that this is similar to their ancestors including their last common ancestor with birds. Birds eventually moved to ZW exclusively as did some fish and insects. Mammals moved to XY exclusively. Reptiles largely lost this completely and depend on temperature. I presume the journal article will provide a lot more answers. Don't forget to consider convergent evolution in any thoughts you have. Also do remember that modern true reptiles, even those considered living fossils like the tuatara are not the last common ancestor of birds, not even close (recent taxonomic and molecular work has shown that they have changed significantly since the Mesozoic era) Nil Einne (talk) 17:36, 6 July 2008 (UTC)[reply]
Evolution of sex may be helpful too Nil Einne (talk) 17:42, 6 July 2008 (UTC)[reply]

The LHC and the end of the world

Recently, someone asked here whether or not the Large Hadron Collider would cause the end of the world. Our answer was, of course, inconclusive but if you're keeping track, you can at least find the minimum value of how much longer we have here. Just FYI...

Atlant (talk) 21:00, 5 July 2008 (UTC)[reply]

Personally, I'm convinced the continued failure of the LHC to go online is a global example of quantum immortality ;-) Someguy1221 (talk) 21:18, 5 July 2008 (UTC)[reply]
Hi. Um, I thought the website said it was going up tomorrow, but now it has apparently been moved to next month, what's going on? Thanks. ~AH1(TCU) 23:40, 6 July 2008 (UTC)[reply]

What the hell is going on here!?

Quantum gravity, quantum black holes, quantum chemistry (?!!). gravity bends time (and is really just acceleration), multiverses, etc., etc., etc. The world of the big and the small is just so mentally.....megawhelming! Please, -someone help me get a grip!--THE WORLD'S MOST CURIOUS MAN (talk) 21:32, 5 July 2008 (UTC)[reply]

Quantum gravity, quantum black hole, quantum chemistry, general relativity, multiverse...You'll really have to ask a more specific question if you want us to help you find an answer. Someguy1221 (talk) 22:04, 5 July 2008 (UTC)[reply]
Quantum theory in general? Fribbler (talk) 22:59, 5 July 2008 (UTC)[reply]
Instrumentalism might make you feel a little more comfortable as a way of reading scientific theories. Or less. I think it was Steve Martin who said that, "College philosophy teaches you just enough to really fuck you up." --Shaggorama (talk) 00:33, 6 July 2008 (UTC)[reply]
From a purely corporate point of view I can tell you that these scientists can "suck my ./..", when a scientific realm has products you can buy then I'd worry if I didn't understand, and maybe take notice of what they were saying.
Hope that helps with your 'megawhelmedness', capitalism loves you, and protects...87.102.86.73 (talk) 16:12, 6 July 2008 (UTC)[reply]
Seriously though there probably is name for the point of view of 'show me something that works', probably not a popular point of view in some circles... Good luck!87.102.86.73 (talk) 16:13, 6 July 2008 (UTC)[reply]
Better not go for an MRI then, you might come face to face with the quantum device that makes it work, then you'll have to come back here and strike-through your post. Franamax (talk) 20:04, 6 July 2008 (UTC)[reply]
Or use GPS, which requires those pesky "gravity bends space and time" corrections. --Stephan Schulz (talk) 22:01, 6 July 2008 (UTC)[reply]
Or use a working device using quantum entanglement for secure distribution of encryption keys. [19] Heck, you might even be able to buy shares someday! Franamax (talk) 23:16, 6 July 2008 (UTC)[reply]

July 6

Colorado 14ers

I am curious about Colorado peaks. How many are there over fourteen thousand feet high? I have seen many different numbers, most of them in the fifties. —Preceding unsigned comment added by 75.169.1.13 (talk) 00:28, 6 July 2008 (UTC)[reply]

Here's the list of them. See also a WP list. Oded (talk) 01:05, 6 July 2008 (UTC)[reply]

Trans Ischemic Attack - same symptoms as stroke in sever cases?

Reading elsewhere, it seems like a transient ischemic attack is only a warning and brain cells don't actually die. Yet, your article speaks of TIAs that can last more than 10 minutes, and seems to say that brain cells do die. So, would the same things as mentioned in stroke apply when it comes to symptoms and treatment, then? In other words, can one experience weakness afterward, maybe periods of aphaxia where one can't talk for a few minutes at a time, etc.? Would rehabilitation be needed even with a TIA or series of them, even if there was no actual stroke that occurred?

Side question - can one be paralyzed on both dies of the body with a TIA/stroke? My hunch is "yes," if in the right place.

I guess it's one of those thigs that will really only be helped once the TIA article grows.209.244.30.221 (talk) 00:35, 6 July 2008 (UTC)[reply]

Yes and yes. TIA is code for "Minor stroke." The key is "transient." A stroke is ischemia caused by a thrombus (blood clot) in the brain that often requires thrombolytic therapy to be corrected and may cause permanent brain damage if not treated in time. The difference between a stroke and a TIA is that in a TIA, the thrombus often dislodges itself before serious damage occurs. Sometimes not. A TIA that causes damage is basically a stroke that treated itself, but a little late. Usually if there's any serious damage, they call it a stroke instead of a TIA. --Shaggorama (talk) 02:22, 6 July 2008 (UTC)[reply]
The difference between a transient ischemic attack and a stroke is the duration. If the symptoms last less than a day, what happened was a TIA; if longer, it was a stroke. They can't be told apart until the 24 hours passes. By definition, though, if the symptoms have disappeared within 24 hours, then there is no residual deficit after that time. A TIA by definition doesn't need rehabilitation, because the neurologic symptoms have disappeared. - Nunh-huh 02:27, 6 July 2008 (UTC)[reply]
This reputable link may help. [20] Richard Avery (talk) 07:39, 6 July 2008 (UTC)[reply]
I liked your link so much I edited the article to reflect the info. Thanks Richard, and don't be afraid to be bold. --Shaggorama (talk) 08:32, 6 July 2008 (UTC)[reply]
Thanks, all, great article; that really helped. —Preceding unsigned comment added by 209.244.187.155 (talk) 10:50, 6 July 2008 (UTC)[reply]

brilliant blue fcf

What is the meaning of fcf in "brilliant blue fcf"? What is fcf stand for in the colouring agents, e.g. in Brilliant Blue FCF, Sunset Yellow FCF. Gcllau (talk) 04:04, 6 July 2008 (UTC)[reply]

food coloring something (anon cmt)
FCF doesn't google all that well. I'm going to guess that it's "Food Colouring Formula". Franamax (talk) 05:11, 6 July 2008 (UTC)[reply]
Wow there isn't much out there, although I vaguely remember having the same question during a chem lab. All I can find is "Food Contact Formulation" (found at the bottom of here). If you search google for that, one of the summaries lists "fast green food contact formulation", and there is indeed a Fast Green FCF. Hope it helps --Bennybp (talk) 05:52, 6 July 2008 (UTC)[reply]

Unexplained Magnetic Phenomenon - Childhood question

Why only, Burnt Matchstick Heads get attracted to a magnet and not unburnt ones. What chemicals are produced after burning that are magnetic in nature and why.
I used to try these scenarios -

1) Matchsticks with heads attached (Wax Paper sticks and Wooden ones).
2) Only Matchstick heads without sticks.
3) Matchstick Heads crushed to fine powder.(To see if weight effects it)

-In all cases only burnt up material got attracted to the magnet.

Regards praneel (talk) 04:19, 6 July 2008 (UTC)[reply]

Unable to confirm. My strike-on-box matches seem to be unmagnetic both before and after they are burned. Perhaps you can be more specific as to brand/type and testing procedure? Dragons flight (talk) 04:34, 6 July 2008 (UTC)[reply]
I forgot the mention this important thing. You have to let the matchstick burn for a while (up to the middle)and gently remove the burnt head from the Burnt Stick with tweezers. Make sure you do it so gently that no burnt wood is left inside the head. This reduces the weight of the head Otherwise it won't work if you put off the flame just after you lite the match.
I went to the market and bought all available brands and did the experiment. Regardless of brand all burnt heads (removed from the burnt stick)are attracted by the magnet. I think I can shoot this video and give that link here. If you want quick results you can remove the head before burning (easy with wax paper matchsticks) or scrape off the powder in chunks as large as possible without any specs of wood. Then burn them.praneel (talk) 05:33, 6 July 2008 (UTC)[reply]
None of the active ingredients in a match head, as listed in our Match article, would become ferromagnetic when burnt, but perhaps your matches contain an impurity such as iron (III) oxide (plain old rust). Iron oxide is not ferromagnetic, so an unlit match containing it would not be attracted to a magnet. However, when a match burns, it releases carbon and carbon monoxide [21], both of which are reducing agents. These would reduce the iron oxide to pure iron, which is ferromagnetic and would be attracted to your magnet. Just a guess. --Heron (talk) 10:51, 6 July 2008 (UTC)[reply]
Note that Gamma ferric oxide is indeed ferromagnetic. It what youve got coating your old cassette tapes! —Preceding unsigned comment added by 79.76.248.187 (talk) 19:09, 6 July 2008 (UTC)[reply]

Evolution of Evolution

  • Can evolution evolve ?
  • Can it evolve faster or slower ?
  • Can it choose to briefly suspend itself ?
  • Can it build on notions of constructivism ?
  • Can it sustain bio-diversity and climate change ?
  • Can evolution explain the golden number ?

- 69.157.240.224 (talk) 04:24, 6 July 2008 (UTC)[reply]

No - no - no - no... All of your questions are based on the terrible (but popular) misconception that evolution has some form of intelligence. Evolution is nothing more than a process. Considering evolution of species, it is a description of how species become different over time primarily based on genetic drift and survival of the fittest. Discussing "evolution of evolution" would require redefining evolution to have some attributes which can evolve. Since evolution doesn't have attributes which evolve, it cannot speed up, pause, or build up any notions. When it comes down to it, your understanding of this will dramatically increase if you look at it from the complete opposite way. Species evolve - even if they are incapable of comprehending what evolution is. The rate at which species evolve is not easy to measure because it happens by chance, not design. Species cannot stop evolving because that would require both a lack of genetic mutation and a perfect fit into the environment that cannot be improved. Species do not evolve on purpose. Giraffes did not get together and vote on how long their necks should be. So, any notion of purpose for evolution is ridiculous. Because species evolve, the total of species changes to better fit the environment. The environment changes, so the species evolve to fit it better. This is ongoing and does not change. The total sum of bio-diversity increases and decreases with environmental changes, but (to my knowledge) has never decreased to one and only one species. Evolution has nothing to do with the golden number. In fact, you can pick any number you like (such as 42) and find that it pops up all over nature. Trying to draw causation from coincidence will lead to nothing but folly. -- kainaw 05:04, 6 July 2008 (UTC)[reply]
Well, on the "faster or slower" question, it depends what you mean by it. Mutations can spread much more rapidly through a population under some circumstances. For example, if the population is very large and has a lot of diversity, mutations don't spread very quickly—you get regression to the mean. If, however, you end up with a population bottleneck, then suddenly your entire population can end up with a given mutation quite rapidly. The idea that evolutionary change happens in "spurts" rather than just gradually at a constant pace is known as punctuated equilibrium. It's not "faster" in the sense you probably mean though—the popular idea that if you "speed up" evolution you'll get some sort of result. What it means is that traits spread to populations quicker. --98.217.8.46 (talk) 14:49, 6 July 2008 (UTC)[reply]
A species can evolve traits that increase or decrease the rate of mutation. It can evolve its preferences for sexual selection. Do those count as evolution evolving? Evolution is required to sustain biodiversity. As for climate change, see Daisyworld. Nothing physical can effect anything mathematical, like the golden ratio. Evolution might be able to explain why people care about the golden ratio, or why it occurs so often in nature, but not the number itself. — DanielLC 15:30, 6 July 2008 (UTC)[reply]
  • Can evolution evolve ? - Sort of, see the folowing responses.
  • Can it evolve faster or slower ? - Yes, both due to changes in the environment and possibly due to changes in the species itself. Sexual reproduction, for example, seems to increase variation in offspring to bring about more rapid adaptation. Some species also appear to have a much high rate of mutation than others. More mutation is better in that it makes for quicker evolution but bad in that more offspring are "defective". Thus, species with many offspring are able to benefit from higher mutation rates without losing a critical number of offspring.
  • Can it choose to briefly suspend itself ? - No.
  • Can it build on notions of constructivism ? - No, although you may have to explain what you mean better here.
  • Can it sustain bio-diversity and climate change ? - If you mean does evolution lead to bio-diversity and species which can survive climate change, then the answer is yes.
If the rate at which evolution occurs can change, "Evolution" doesn't consciously change that rate. That's a bit like saying that "Rain" consciously decides whether to rain more heavily or not.
I blame the creators of Heroes and X men (and the like) for giving children a false impression of what evolution is. I wouldn't mind if creators of fiction were banned from using the word "evolution" unless they used it to describe Biological Evolution as explained in that article. Zain Ebrahim (talk) 12:42, 7 July 2008 (UTC)[reply]
That is why I said "no, no, no..." Evolution is nothing more than a word used to describe a process. It is not a living, thinking, planning thing. It has no speed. It has no purpose. This question is no different than asking "Does the Dewey decimal system speed up or slow down?" The only way to provide an answer is to redefine evolution as having some sort of speed or attribute of evolution itself. Once you redefine evolution, you are no longer talking about evolution. -- kainaw 12:59, 7 July 2008 (UTC)[reply]

Unleaded fuel

How can the effects of using leaded fuel or LRP in a car that is supposed to use unleaded fuel be overcome —Preceding unsigned comment added by Razzmetazz (talkcontribs) 05:34, 6 July 2008 (UTC)[reply]

Feel free to ignore this if you want but may I ask why? According to Tetra-ethyl lead "and the only countries in which leaded gasoline is extensively used are Yemen, Afghanistan and North Korea[citation needed]." I presume if you are living in Yemen, Afghanistan or North Korea, you probably have a car which is not designed to use leaded fuel. In any case Catalytic converter doesn't mention any methods to prevent fouling with lead. My gut feeling is it's difficult if possible. More likely you'd have to use a significantly different catalytic converter which is pointless since you might as well just switch to unleaded fuel and if you can't, e.g. because your Afghanistan your vehicles are probably already so polluting that equiping them with catalytic converters is somewhat pointless, there's more effective things to do first (e.g. switch to unleaded fuel). Presuming you've made the mistake of using leaded fuel in a car with a catalytic converter, our article mentions "Depending on the contaminant, catalyst poisoning can sometimes be reversed by running the engine under a very heavy load for an extended period of time. The increased exhaust temperature can sometimes liquefy or sublime the catalytic contaminant, removing it from the catalytic surface. However, removal of lead deposits in this manner is usually not possible due to lead's high boiling point. In particularly bad cases of catalytic lead poisoning, the catalytic converter may actually become completely plugged with lead residue". My suggestion would be to just replace the catalytic converter... Nil Einne (talk) 17:21, 6 July 2008 (UTC)[reply]
If you're going to burn fuel with lead in it, you might as well replace the catalytic converter with a piece of regular exhaust pipe too. But would there be additional concerns with fouling of the various mass-flow sensors in the engine-management system? Franamax (talk) 18:03, 6 July 2008 (UTC)[reply]
I'm going to make an educated guess here that you may have based your question on incorrect info. Many years ago gasoline grades in the US were called "Regular", "Unleaded", and "Premium". At that time the "Regular" and "Premium" both meant leaded gasoline. However, at some point it became illegal to sell leaded gasoline in the US, at which point the grades were called "Regular", "Plus", and "Premium" (the names vary slightly from brand to brand). The "Regular" and "Premium" now have a different meaning, as they, along with "Plus", refer to unleaded gasoline only. However, I could certainly see how someone might think that, since the grade called "Unleaded" had disappeared, that this meant that unleaded gasoline was no longer sold. StuRat (talk) 18:51, 6 July 2008 (UTC)[reply]
It is possible that the questioner is making a false assumption based on good knowledge. If you have a car that is designed to run on leaded gasoline, using unleaded gasoline will reduce the effective octane level a tiny bit (about 85 instead of 87 octane). It isn't enough to really notice. Now, if you assume this applies in reverse, you will think that you can take a car designed for unleaded gasoline, put leaded gasoline in it, and get an effective boost in the octane level. Sorry, but that isn't true. All you will do is throw off the engine's timing (effectively lowering the octane level) and increase lead pollution in the air. The key is understanding that the engine is timed for the specific type of gasoline it is intended to use. If you use a different type of gasoline, the timing will be off. You won't use the combustion of the gasoline to its full potential and, in the end, run at what could be effectively be referred to as a lower octane level. The gas doesn't actually lose octane. The engine just doesn't use it properly. -- kainaw 00:05, 7 July 2008 (UTC)[reply]

Courage

What is the most coragious felin? And the most coragious animal? Francesco —Preceding unsigned comment added by 87.20.213.78 (talk) 08:25, 6 July 2008 (UTC)[reply]

That's difficult to answer - how do you distinguish bravery and stupidity? There are plenty of animals that wouldn't be scared of something dangerous simply because it's not something they would ever encounter in their natural environment, and have never encountered before. That's not bravery, it's just ignorance. --Tango (talk) 13:19, 6 July 2008 (UTC)[reply]
Most courageous animal? Must be deers. When faced with several tons of steel running toward them at 70 mph, most deers would choose to stand their ground instead of running. Never have I seen another species display such gallantry so consistently :) --71.175.20.73 (talk) 13:48, 6 July 2008 (UTC)[reply]
Same with several hundred tonnes at 100 mph. An antler charge on an intercity train... that was brave (and messy). --BozMo talk 14:43, 6 July 2008 (UTC)[reply]
In general, many female animals with young display incredible bravery when threatened. Courage is probably related to fighting skill, I'd nominate polar bears and especially wolverines, which even bears don't mess with. Franamax (talk) 17:59, 6 July 2008 (UTC)[reply]
I'd disagree , surely courage is measured in part by the odds against you, meaning that flys/ants etc are extremely brave (and with a life expectency to match...)87.102.86.73 (talk) 19:40, 6 July 2008 (UTC)[reply]
Yes, there was a case of a house cat which rescued it's kittens from a burning building, one at a time, and burned both of it's ears off in the process. That's quite the act of bravery. StuRat (talk) 18:43, 6 July 2008 (UTC)[reply]
Personally, I'm not sure I'd ascribe "courage" to most animals at all. I tend to think of courage as a willingness to face danger/pain/struggle, but that presupposes an awareness and anticipation of the danger/pain/etc. If an animal acts on instinct, without awareness of the potential consequences of its actions, then for me those actions don't seem courageous. I'm sure there are exceptions for some actions by some more intellectually capable animals, but to the degree that much of animal behavior is instinct devoid of foresight or planning, then I'd see those actions shouldn't be interpreted as courageous at all. Dragons flight (talk) 18:54, 6 July 2008 (UTC)[reply]
Yes, we are antropomorphizing here. Blame it on that documentary about the penguins. However sometimes what we call courage could also be interpreted as stupidity, a lot of courageous acts are performed by humans acting on what you could call instinct, i.e. insufficient assessment of the risks. It seems to me that at least some of the people who rescue someone from a smashed-up car in a pool of gasoline must reflect later and think "what the hell was I doing?". Franamax (talk) 19:56, 6 July 2008 (UTC)[reply]
And, instinct or not, a mother facing down a predator to protect her young satisfies your criterion of "willingness to face danger" with "awareness...of the danger". Franamax (talk) 19:59, 6 July 2008 (UTC)[reply]


I don't feel that an animal standing it's ground, like a deer as described above, is *necessarily* brave. Some animals become paralysed with fear. This fear may be misconstrued as bravery or courage.

As for the most courageous animal, the well-bred German Shepherd dog. But then, I'm a bit biased.

I think most people tend to give animals human emotions. A silly practice if you ask me.


Ruairí Óg the Rogue (talk) 05:13, 7 July 2008 (UTC)[reply]

Life in free-fall

Hi, how would menstruation work in free-fall? At first I thought tampons might work, but then you can't reach the uterine wall with a tampon.

Also, does gravity play a roll in how a fetus is attached to the uterine walls? Any help would be much appreciated. Thanks. --Kjoonlee 14:23, 6 July 2008 (UTC)[reply]

According to various google hits ([22]), gravity is not required for menstruation, so it happens with no difficulty in space. I have no idea how it works, though. [23] says that pregnancy in space appears to be possible. --Tango (talk) 15:46, 6 July 2008 (UTC)[reply]
First, tampons aren't inserted into the uterus, just the vagina. They would have no problem reaching the vaginal wall since they expand when wet, and this works fine without gravity. The only concern is that gravity may be important in removing the menstrual fluid quickly, before it starts to decompose. I imagine bedridden women have faced this issue before, however. Menstruation obviously still "works" in zero-g, but it may be a bit less efficient, meaning women need to be extra careful not to do other things which might make the situation worse, like leaving the same tampon in too long. They may be more susceptible to toxic shock syndrome in zero-g than normal gravity, if they aren't careful. StuRat (talk) 18:36, 6 July 2008 (UTC)[reply]
Menstruation is probably less problematic in space than when bed-ridden because women in space can move around. This is important if period pains are to be avoided. Itsmejudith (talk) 20:59, 6 July 2008 (UTC)[reply]

Iron lattice

The lattice spacing of iron given at [24] is 286.66 pm. Can this be calculated? How? Thanks, *Max* (talk) 18:26, 6 July 2008 (UTC).[reply]

Typically experimentally by X-ray crystallography
Alternatively: knowing the density of the solid, the atomic mass, and the number of atoms per mole, and assuming a crystal structure you can make a good estimate with out using x-rays..
Please ask if you require further information on either method.87.102.86.73 (talk) 19:38, 6 July 2008 (UTC)[reply]
I wanted to know how it could be calculated from the electromagnetic forces between the atoms. Is this possible? *Max* (talk) 03:26, 7 July 2008 (UTC).[reply]
You can certainly do molecular modelling calculations to minimize the energy for a collection of iron atoms. For your purposes, you would want some sort of ab initio quantum chemistry methods, as any of the simple molecular mechanics or Semi-empirical quantum chemistry method approaches would likely be based on observed interatomic distances, so you wouldn't really be calculating them. DMacks (talk) 04:09, 7 July 2008 (UTC)[reply]
You could try to get an estimate this way.. even using classical inverse square electric field. - but a computer would help a lot - simply place the charged iron nucleii in the desired lattice and add the electrons randomly to the lattice.. then let the electrons and nuclei move under the resultant forces (do this stepwise) until some sort of equilibrium is found... NOTE you need to introduce a abitrary barrier around the nucleus to prevent the electrons falling into the nucleus (ie a bounding spherical shell within which the electrons are not allowed to go)..
You can repeat this experiment with smaller and smaller bounding spherical shell and maybe even calculate the limit as the shell radius tend to zero. (you can also replace electrons with N pseudo-electrons of charge e/N to get a better approximation to electron density) This I suspect would work quite well...
I can't think of a method that you could easily do on paper though.. not without resorting to heavy maths or a heavyily simplified model.87.102.86.73 (talk) 13:17, 7 July 2008 (UTC)[reply]

Flashes from Zen Vision W

The flashes from my Zen Vision W, which I told you about earlier, are real, not my imagination. I've just seen one while looking directly at the device. The screen flashed a solid light gray for about a tenth of a second, while the device was turned off. What is the cause of this? JIP | Talk 17:33, 5 July 2008 (UTC)[reply]

So the backlight turned on by itself? --antilivedT | C | G 23:43, 5 July 2008 (UTC)[reply]
Seems like that, yes. And it turned off by itself immediately afterwards. JIP | Talk 18:18, 6 July 2008 (UTC)[reply]

Redirected from Wikipedia:Reference desk/Miscellaneous#Flashes from Zen Vision W87.102.86.73 (talk) 19:33, 6 July 2008 (UTC)[reply]

For the record the previous question was here Wikipedia:Reference_desk/Miscellaneous#Weird_flashes_from_Zen_Vision_W, it seems the light is real, and not imagined.. what causes this.?87.102.86.73 (talk) 19:33, 6 July 2008 (UTC)[reply]
I'm not sure about Zen Vision W, but most media players feature only a soft-off power switch. This means that even when the device is in a power-off state, the internal circuitry is still on, but consumes only a very small amount of electricity. So, these flashes could be caused by a faulty component, broken connection, or even a firmware bug. You should first check whether these flashes still happen with the battery removed. —Preceding unsigned comment added by 85.222.93.28 (talk) 00:27, 7 July 2008 (UTC)[reply]
I doubt the battery is removable (without voiding the warranty, that is). Secondly, what would that prove? I think I can pretty much guarantee that if you remove the battery, it's not going to do any flashing -- no power, no light, surely? -- Captain Disdain (talk) 11:51, 7 July 2008 (UTC)[reply]
What about somesort of residual charge in the light driver (note I don't actually know if its fluorescent source or Electro-luminesencent..)
I also think I've seen a similar effect say ~15mins? after a electronic fluorescent light has been switched off (see linked original question), additionally I vaguely remember a similar effect from old black and white TV's after they had been switched off.. I suggested some sort of cascade process... but where would the residual voltage come from?? and what would cause the electron cascade to start: cosmic rays? 87.102.86.73 (talk) 13:09, 7 July 2008 (UTC)[reply]

Citing

Hello. I am making a registry of the most commonly cited sources that I use. When citing in APA style, must I include the author's title (i.e., Dr.) as part of the first name initial? For example, would the following be correct?

Jenkins, Dr F., Van Kessel, H., Davies, L., Lantz, Dr O., Thomas, P., & Tompkins, D. (2002). Chemistry 11. Canada: Nelson Thomson Canada Limited.

Thanks in advance. --Mayfare (talk) 20:07, 6 July 2008 (UTC)[reply]

Almost certainly not, I can't recall ever seeing a 'Dr.' title in a citation..87.102.86.73 (talk) 20:13, 6 July 2008 (UTC)[reply]
No, you don't use titles at all, much less professional ones. --98.217.8.46 (talk) 20:15, 6 July 2008 (UTC)[reply]

Breeding spiders

My jumping spider, (I think Dendryphantinae) has spun a nest, and laid her eggs. The spiderlings have hatched but are still in the nest (they can't/don't come out). When raising spiderlings should they be taken out of the nest or left on their own until they come out themselves? (Maybe someone who has raised Tarantulas can tell me what is done in that case). Pro bug catcher (talkcontribs). 21:41, 6 July 2008 (UTC)[reply]

I'm not a spider expert but it seems unlikely to me that human intervention would be helpful in this situation. The spiders aren't waiting for you. They are probably only marginally aware of your existence. --98.217.8.46 (talk) 21:46, 6 July 2008 (UTC)[reply]
There are many (too many) spiders that make my yard their home. I often see nests teeming with tiny legged specks moving all around. After a while, they come out, sling out a little sting of web, and float off. Then, I have dozens of them trying to make a new home on my car. -- kainaw 23:57, 6 July 2008 (UTC)[reply]

July 7

what kind of snake?

Does anyone know what kind of snake this is? I spotted it in the forest near Multnomah Falls in Oregon, United States. Shown with a US nickel. Thanks! Dar-Ape 01:12, 7 July 2008 (UTC)[reply]

What percent of heart attacks go undetected each year?

I'm not talking about ones where the person dies, and they just guess; I mean, is there some estimate of how many people might have really mild ones and not know it. I read that Bo Schembechler likely had one. I suspect it's someone common in high stress jobs to just think you're having angina when in fact you're having an actual heart attack.

If untreated, would the person who had one just recover naturally, or would there still be problems? I guess if they 'slow it down" for a few weeks then are back at it, they could for a while, right? Or, would an untreated, yet mild, heart attack still cause problems later? A friend I know thinks they might have had one, though they thought at the time it was angina; they eventually went into another field that was less stressful, and any bouts with shrotness of breath left.209.244.30.221 (talk) 01:15, 7 July 2008 (UTC)[reply]

I don't know enough about the topic to comment on the rest of your questions, but I'm pretty sure that if someone dies of a heart attack, they don't have to guess: a post mortem medical examination is going to show that as the cause of death pretty conclusively. (Also, if your friend thinks they might've had a heart attack, they should go and see a doctor. Frankly, I'm a little shocked if they haven't figured this much out by themselves. I mean, exactly what does it take for them to start to consider seeing a doctor if that doesn't do it?!) -- Captain Disdain (talk) 01:48, 7 July 2008 (UTC)[reply]
I know people say this all the time, but Wikipedia will not give you medical advice. If your friend thinks he has a medical problem he ought to consult a licensed physician not a bunch of anonymous folks on the internet, brilliant though we are. That said, I doubt the statistic you are looking for exists. By definition, data is not collected on heart attacks that go undetected, so it is very hard to compile information on them. Plasticup T/C 02:56, 7 July 2008 (UTC)[reply]
Up to ~40% of heart attacks are not recognized [25]. Any heart attack has the potential to cause lasting damage. Looking for that damage is in fact the easiest way to detect the signs of a heart attack in patients who had an episode and didn't recognize it. If your friend did have a heart attack, there is a good chance that an EKG would be able to confirm, and as always we'd encourage you to speak to a doctor. Dragons flight (talk) 03:37, 7 July 2008 (UTC)[reply]
Neat study. Thanks DF. --Shaggorama (talk) 04:32, 7 July 2008 (UTC)[reply]

Similiar heartbeats

Is it possible for tow unrelated people to share very similar cardiac patterns for a prolonged period of time. 69.157.226.161 (talk) 01:49, 7 July 2008 (UTC)[reply]

I would imagine any healthy people are likely to have similar cardiac patterns. Any difference from the norm usually indicates a problem. --Tango (talk) 01:55, 7 July 2008 (UTC)[reply]
The resting heart rate is similar amongst humans, though it depends on body size and shape. You're heart rate is also constantly changing depending on your current level of activity. It is possible for your heart rate to appear to match someone else's exactly, but only briefly. --Russoc4 (talk) 02:44, 7 July 2008 (UTC)[reply]
Heart rhythms are so similar that EKG machines are tuned to pick up very minor fluctuations. I have a slightly long Q wave by 2 milliseconds (usually) and it is always picked up on an EKG. So, if you aren't looking into the millisecond range for differences in heart rhythms, you'll likely see nearly identical rhythms running at different rates (ie: 70 beats a minute compared to 80 beats a minute). -- kainaw 04:27, 7 July 2008 (UTC)[reply]

Can vets tell what Animal species

Media:Example.ogg —Preceding unsigned comment added by 98.203.9.163 (talk) 05:31, 7 July 2008 (UTC)[reply]

Hertz.

What is the difference between 50Hz and 60Hz?

Could a 60Hz supply damage a 50Hz appliance? —Preceding unsigned comment added by 41.241.32.81 (talk) 06:09, 7 July 2008 (UTC)[reply]

Hertz is a measure of frequency. In this case, it describes the number of times that an AC current changes polarity per second. It's effect depends on the appliance, but in principle yes, it could. More important is the difference in voltage that often goes with the difference in frequency. Some appliances will not work at all, some at a different speed, and some may be damaged. Modern electronic devices usually have power supplies that accept a range of voltages and frequencies (50-60Hz, 100-240V nominal). Check your appliance for a type sign. --Stephan Schulz (talk) 10:58, 7 July 2008 (UTC)[reply]
If you say the type of appliance then someone here will probably know whether it is likely to be damaged or not. Itsmejudith (talk) 11:01, 7 July 2008 (UTC)[reply]

chemitry

why are most types of glassware use in a chemical laboratory made of pyrex? —Preceding unsigned comment added by 125.60.237.218 (talk) 13:14, 7 July 2008 (UTC)[reply]

It's not always 'pyrex' though most borosilicate glasses are often refered to as pyrex.. the reason is its thermal shock resistance..
I recommend reading both the links pyrex and borosilicate glass.87.102.86.73 (talk) 13:25, 7 July 2008 (UTC)[reply]
note a lot of specialised lab glass ware will be made out of quartz, but only very specialised stuff.87.102.86.73 (talk) 13:20, 7 July 2008 (UTC)[reply]

Need Help Identifying Scary and Fast Bug.

Every once in a while, before I turn on the water to take a shower, a bug that looks really hairy or has a thousand legs darts out of nowhere and scares the poop out of me. It is realy, really fast and I am not around a camera to take a picture. Do you know what kind of bug this is? BTW - I live in New England, USA. --Anthonygiroux (talk) 13:17, 7 July 2008 (UTC)[reply]

"Universe in My Thumb"

Someone told me that there is (or could be) a whole universe in my thumb. Do you know what they mean by that? --Anthonygiroux (talk) 13:19, 7 July 2008 (UTC)[reply]

I personally have no idea. Maybe it was an idiom? — CycloneNimrod  Talk? 13:24, 7 July 2008 (UTC)[reply]

How could this be?

Tomoji Tanabe is 112 and seems to be extremely healthy. My grandmother is 86 and is in hospital sick. Higher ages seem to be more common in females, so how was this jumble caused? 208.76.245.162 (talk) 13:31, 7 July 2008 (UTC)[reply]